You are on page 1of 28

EMERGENCY MEDICINE

.
PRACTICE EMPRACTICE NET
A N E V I D E N C E - B A S E D A P P ROAC H T O E M E RG E N C Y M E D I C I N E

August 2002
Atrial Fibrillation In The ED: Volume 4, Number 8

Cardioversion, Rate Control, Author

Anticoagulation, And More Gary F. Pollock, MD, FACEP


Clinical Instructor of Emergency Medicine,
University of Pittsburgh Affiliated Residency in
Emergency Medicine; Assistant Education Director,
The man in bed 2 is pale and diaphoretic. The monitor shows a heart rate of 160 and a Trauma Liaison, The Mercy Hospital of Pittsburgh,
blood pressure of 70. The nurse looks at you and says, “Doctor, what should we do?” Pittsburgh, PA.
The elderly lady in the next bed is yelling, “Doctor, my chest hurts!” Her monitor tells
Peer Reviewers
all—an irregular wide complex rhythm at a rate of 130. You ponder what drug to give.
Just then, the medics break through the door with yet another patient with a fast heart Mel E. Herbert, MD, MBBS, BMedSCi, FACEP
rate. The adenosine they administered didn’t help. Assistant Professor, Medicine, UCLA School of
As the confusion and pressure mount, you hear a faint shriek of an alarm—first, Medicine; Assistant Professor, Nursing, UCLA
School of Nursing, Department of Emergency
distant; then, more insistent. You wake. Thank goodness, it was only a dream. As you
Medicine, Olive View-UCLA Medical Center,
walk into the ED for your morning shift, the charge nurse grabs your arm and says, Sylmar, CA.
“The guy in bed 2 really needs you…his heart rate is 160.”
Corey M. Slovis, MD, FACP, FACEP
Professor of Emergency Medicine and Chairman,
A TRIAL fibrillation (AF) and its close counterpart, atrial flutter (AFl),
are two of the most common arrhythmias encountered in emergency
medicine. Their wide spectrum of presentation can challenge even the savviest
Department of Emergency Medicine, Vanderbilt
University Medical Center; Medical Director, Metro
Nashville EMS, Nashville, TN.
clinician. Management concerns encompass diagnostic studies, rate control,
cardioversion, anticoagulation, and disposition. CME Objectives
The emergency physician must understand the complexity of these issues
Upon completing this article, you should be able to:
in order to manage the diverse presentations of AF. In most cases, AF and AFl 1. discuss the evaluation and management of new-
can be discussed as one entity; however, there are important differences that onset atrial fibrillation in the ED;
should be understood. This issue of Emergency Medicine Practice addresses the 2. list the common rate control agents and explain each
emergent complications of AF and AFl in an evidence-based fashion and drug’s indications and contraindications;
3. explain how and when it is appropriate to start
provides a measured approach to this often “irregular” topic.
anticoagulation in a patient with AF; and
4. discuss admission criteria and appropriate disposition
Critical Appraisal Of The Literature of patients with new-onset atrial fibrillation based on
the current literature.
Numerous studies regarding AF are pertinent to its evaluation and treatment
in the ED. There are very few studies that specifically address AF in the Date of original release: August 1, 2002.
prehospital setting, and none of them are randomized. Date of most recent review: July 11, 2002.
Of the numerous studies that evaluate rate control and conversion agents, See “Physician CME Information” on back page.

Editor-in-Chief Center School of Medicine, Emergency Medicine, Morristown Professor and Chief of the Division Medical Service, Orlando, FL.
Albuquerque, NM. Memorial Hospital. of Family Medicine, Mount Sinai Alfred Sacchetti, MD, FACEP,
Stephen A. Colucciello, MD, FACEP, School of Medicine, New York, NY. Research Director, Our Lady of
W. Richard Bukata, MD, Assistant Michael A. Gibbs, MD, FACEP, Chief,
Assistant Chair, Department of Clinical Professor, Emergency Department of Emergency John A. Marx, MD, Chair and Chief, Lourdes Medical Center, Camden,
Emergency Medicine, Carolinas Medicine, Los Angeles County/ Medicine, Maine Medical Center, Department of Emergency NJ; Assistant Clinical Professor
Medical Center, Charlotte, NC; USC Medical Center, Los Angeles, Portland, ME. Medicine, Carolinas Medical of Emergency Medicine,
Associate Clinical Professor, CA; Medical Director, Emergency Center, Charlotte, NC; Clinical Thomas Jefferson University,
Department of Emergency Gregory L. Henry, MD, FACEP,
Department, San Gabriel Valley Professor, Department of Philadelphia, PA.
Medicine, University of North CEO, Medical Practice Risk
Medical Center, San Gabriel, CA. Emergency Medicine, University Corey M. Slovis, MD, FACP, FACEP,
Carolina at Chapel Hill, Chapel Assessment, Inc., Ann Arbor,
Francis M. Fesmire, MD, FACEP, MI; Clinical Professor, Department of North Carolina at Chapel Hill, Professor of Emergency Medicine
Hill, NC. Chapel Hill, NC.
Director, Chest Pain—Stroke of Emergency Medicine, and Chairman, Department of
Associate Editor Center, Erlanger Medical Center; University of Michigan Medical Emergency Medicine, Vanderbilt
Michael S. Radeos, MD, MPH,
Assistant Professor of Medicine, School, Ann Arbor, MI; President, University Medical Center;
Attending Physician, Department
Andy Jagoda, MD, FACEP, Professor UT College of Medicine, American Physicians Assurance Medical Director, Metro Nashville
of Emergency Medicine,
of Emergency Medicine; Director, Chattanooga, TN. Society, Ltd., Bridgetown, EMS, Nashville, TN.
Lincoln Medical and Mental
International Studies Program, Barbados, West Indies; Past Health Center, Bronx, NY; Mark Smith, MD, Chairman,
Valerio Gai, MD, Professor and Chair,
Mount Sinai School of Medicine, President, ACEP. Assistant Professor in Emergency Department of Emergency
Department of Emergency
New York, NY. Medicine, University of Turin, Italy. Jerome R. Hoffman, MA, MD, FACEP, Medicine, Weill College of Medicine, Washington Hospital
Professor of Medicine/Emergency Medicine, Cornell University, Center, Washington, DC.
Michael J. Gerardi, MD, FACEP,
Medicine, UCLA School of New York, NY.
Editorial Board Clinical Assistant Professor, Charles Stewart, MD, FACEP,
Medicine; Attending Physician, Colorado Springs, CO.
Medicine, University of Medicine Steven G. Rothrock, MD, FACEP, FAAP,
UCLA Emergency Medicine Center;
Judith C. Brillman, MD, Residency and Dentistry of New Jersey; Associate Professor Thomas E. Terndrup, MD, Professor
Co-Director, The Doctoring
Director, Associate Professor, Director, Pediatric Emergency of Emergency Medicine, University and Chair, Department of
Program, UCLA School of Medicine,
Department of Emergency Medicine, Children’s Medical of Florida; Orlando Regional Emergency Medicine, University
Los Angeles, CA.
Medicine, The University of Center, Atlantic Health System; Medical Center; Medical Director of of Alabama at Birmingham,
New Mexico Health Sciences Vice-Chairman, Department of Francis P. Kohrs, MD, MSPH, Associate Orange County Emergency Birmingham, AL.
most are small and have differing methodology, which bars acute myocardial infarction (MI).7 Idiopathic and cardiac
easy comparison. Outcomes vary greatly depending on causes (such as hypertensive heart disease, valvular disease,
patient characteristics, drug selection, dose, and mode of CHF, and coronary artery disease) account for the largest
delivery. The fact that the duration of AF differs tremen- percentage of AF.4,8 Thyroid disease is another important
dously between trials significantly affects outcomes. consideration;9 in one series, it was responsible for 6.2% of
The American College of Cardiology, American Heart new-onset atrial fibrillation (NOAF) cases admitted to the
Association, and the European Society of Cardiology, in hospital.8 Moderate-to-severe hypothermia (< 32°C) is
collaboration with the North American Society of Pacing frequently associated with AF.10 Electrolyte abnormalities
and Electrophysiology, have published a summary report of (hypokalemia and hypomagnesemia), as well as drug and
practice guidelines for the management of patients with AF.1 medication use (especially cocaine and theophylline
This set of guidelines is evidenced-based and well done; toxicity) can produce AF. Medical noncompliance in patients
however, it largely tends to focus on the management of with known AF accounts for a large percentage of cases of
AF/AFl outside the ED. rapid AF seen in the ED.
The newest version of the ACLS guidelines appears to “Holiday heart syndrome” is a colorful term applied to
be more evidenced-based with respect to recommenda- arrhythmias resulting from alcohol abuse, AF being the
tions.2 The ACLS algorithm for the management of atrial most common presentation of this often-seasonal disorder.
fibrillation is complex. Patients are classed according to Lowenstein et al found that alcohol caused or contributed to
heart function (something not always known in the ED), 35% of 40 cases of NOAF at a public hospital.11 Holiday
and treatment options depend partly on the duration of the heart syndrome is most often reported in the setting of
AF. Use of the guideline may be within the standard of care alcohol intoxication, but it may be caused by mild alcohol
for an ED physician, but it may be somewhat too complex withdrawal as well.11 The mechanism of how alcohol causes
for other providers who may also manage these patients. AF is unclear. Many of these cases spontaneously convert to
normal sinus rhythm within 24 hours.11 Holiday heart
Epidemiology syndrome should not be ascribed to the older patient or
those with cardiac disease until other etiologies have been
The incidence of AF increases with age and approximately ruled out.
doubles with each decade of adult life. This incidence will
likely continue to increase as the population ages.3 The “A good head and a good heart are always a formidable
prevalence of AF in the adult population has been reported combination.”—Nelson Mandela
to be 1%-4%, with up to 9% of patients over the age of 80
having AF.3,4 Women appear to have a slightly lower Pathophysiology
prevalence than men, while it is uncommon in children
except after cardiac surgery.1 AF results from chaotic depolarization of atrial tissue. The
AF is the most common sustained arrhythmia in exact cause is not entirely clear, but multiple reentry circuits
medicine and is one of the most common arrhythmias seen or rapidly firing atrial foci are likely involved.1 The mecha-
in the ED. It is associated with significant morbidity, nism most commonly reported in AFl involves a large
including embolic strokes, decreased exercise tolerance, and reentrant circuit within the right atrium around an anatomic
tachycardia-induced myopathy. Mortality rates are doubled obstacle. Atrial distention contributes to AF, particularly
in patients with chronic AF compared with controls.4 The with certain types of valvular disease. There is a saying that
annual risk of stroke in patients with AF averages 5%, which “AF begets AF,”12 meaning that the longer an individual is
is 2-7 times the rate in people without AF. According to data in AF, the more likely he or she will stay in AF. This appears
from the Framingham cohort, stroke severity tends to be to be the result of mechanical and electrical remodeling in
greater in patients with AF than those without the disease.5 the atria. Atrial remodeling in AF appears to occur quickly,
AFl is also a common arrhythmia. It is most often
associated with cardiovascular and pulmonary disease, but Table 1. Etiology Of Atrial Fibrillation.
it can occur in those with normal hearts. AFl is associated
with atrial fibrillation in more than half the cases, and both Cardiac Non-cardiac
of the rhythms can be present on the same ECG (fib-flutter). • Ischemic heart disease • Pulmonary embolism
• Valvular disease • Idiopathic
Like AF, its incidence increases with age. AFl is associated
• Hypertension • Medication
with thromboembolic events, but probably to a lesser extent
• Congestive heart failure noncompliance
than is seen with fibrillation.6 • Sick sinus syndrome • Thyroid disease
• Pericarditis • Holiday heart syndrome
Etiology • Infiltrative heart • Medication use
disease • Electrocution
The common causes of atrial fibrillation are listed in Table 1. • Cardiomyopathy • Other pulmonary disease
The emergency physician should first consider immediate • Cardiac surgery • Chest trauma
and life-threatening causes, such as cardiac ischemia, • Myocarditis • Hypokalemia
congestive heart failure (CHF), and pulmonary embolism • Congenital heart • Hypomagnesemia
(PE). AF occurs in up to 20% of patients who present with disease • Hypothermia

Emergency Medicine Practice 2 www.empractice.net • August 2002


is progressive, and may be persistent. However, it can be rhythm’s underlying cause? The emergency physician must
reversed with early restoration and maintenance of sinus always consider the “worst first,” which, in the case of atrial
rhythm.13 While mechanical dysfunction of the atria can fibrillation, would include PE (and other causes of hypoxia),
occur with prolonged AF, restoration of sinus rhythm coronary ischemia, and CHF. Additionally, if the AF is chronic,
will generally restore atrial function. This process can take high ventricular rates may be due to fever, blood loss, stress,
2-4 weeks, during which time there is an increased risk dehydration, or drug toxicity.
of thromboembolism.14
In AF and AFl, the atrioventricular (AV) node plays a Prehospital Care
crucial role in modulating the ventricular response. The AV
nodal refractory period, the amount of concealed conduc- The prehospital care of a patient starts with an evaluation of
tion within the node, the relative degree of sympathetic the ABCs. The airway should be assessed and stabilized,
tone, and the level of circulating catecholamines all interact followed by an evaluation of the patient’s respiratory and
to determine the ventricular rate. The untreated heart rate in circulatory system. A rapid pulse should prompt care
a patient with NOAF will usually be 120-160 beats per providers to place the patient on the monitor and analyze
minute (bpm). In AFl, ventricular rates are usually around the rhythm. An unstable patient with atrial fibrillation who is
150 bpm due to 2:1 conduction of flutter waves through the severely compromised needs emergent synchronized cardioversion.
AV node. The consequences of the resultant tachycardia in If the patient is stable, then a brief history and physical
untreated AF/AFl will vary depending on the ventricular examination should be performed during transport. Medics
rate, underlying heart disease, duration of AF, co-morbidi- should apply oxygen (or maintain saturations at 94%
ties, and other patient factors. or above), establish an IV, and institute continuous
The spectrum of physiologic effects of a rapid ventricu- ECG monitoring.
lar response (rapid AF) is broad, ranging from asymptom- The evidence for the management of AF in the field is
atic to severe, life-threatening symptoms such as CHF, mostly based on data from the hospital, as there are very
syncope, and angina.15 A rapid ventricular response rate few prehospital studies of this condition.19-22 Current ACLS
increases oxygen demand and decreases diastolic filling guidelines recommend IV calcium-channel blockers or ß-
time. Patients with underlying coronary artery disease may blockers as first-line therapy in stable patients with rapid
develop ischemia during periods of rapid ventricular rate. AF.2 However, a recent small, retrospective study ques-
In addition to rate-related problems in AF, the loss of the tioned the utility of prehospital rate control of rapid AF. The
“atrial kick” may lead to a significant decrease in cardiac study concluded that prehospital rapid AF is an infre-
output. This is especially prominent in those with vascular quently encountered, predominantly hemodynamically
disease and in the elderly, in whom 30% of the cardiac stable cardiac arrhythmia that is readily stabilized using
output may depend on the “atrial kick.” symptomatic or supportive care.19 This study suggests that
Stagnant blood in flow through the atria can produce the use of IV diltiazem for the treatment of rapid AF in the
cardiac clots and subsequent embolism. Stroke is often the field might be unnecessary.
first presentation of AF.8,16 Atrial flutter, despite it being a Given the data currently available, it is hard to recom-
more organized process, also carries a risk of embolic mend universal use of prehospital agents to control rates for
events.17 Intra-atrial clot has been documented by rapid AF. Most patients appear to do well with basic
transesophageal echocardiography in up to 21% of selected monitoring, oxygen, and supportive care. Certainly, special
hospitalized patients with AFl.18 The risk of clot develop- circumstances (e.g., long transport times, moribund
ment in AFl can be due to the flutter itself or the fact that AF patients) may dictate more aggressive interventions.
and AFl often coexist.6,17
Emergency Department Evaluation
Differential Diagnosis The management of uncontrolled atrial fibrillation involves
When considering the differential diagnosis, the emergency recognition, stabilization, diagnostic testing, and disposi-
physician must consider two important questions. First, is tion. Part of stabilization may involve ventricular rate
this atrial fibrillation, atrial flutter, or some other arrhyth- control. Diagnostic testing should avoid needless studies yet
mia? Misdiagnosing ventricular tachycardia (VT) or Wolff- at the same time detect significant pathology. Because AF
Parkinson-White (WPW) syndrome as AF/AFl can be very can lead to stroke and other forms of thromboembolism, the
dangerous. VT is a regular wide complex rhythm, whereas emergency physician must understand the appropriate
AF with a bundle branch block will demonstrate gross (or timing and role of anticoagulation and initiate this therapy
occasionally subtle) irregularity when evaluated closely. when appropriate. There is some recent evidence to suggest
(WPW with AF is discussed in detail under the “Special that some patients with NOAF can be converted in the ED
Circumstances” section later in this article.) Physicians often and discharged.23,24
mistake multifocal atrial tachycardia for AF, as it often “We are adhering to life with our last muscle—the heart.”
appears irregular. However, the presence of discreet P —Djuna Barnes
waves of varying morphology and PR intervals will make
the diagnosis. Initial Stabilization
The second question to entertain is, what is the Immediately assess the unstable patient to determine the

August 2002 • www.empractice.net 3 Emergency Medicine Practice


need for airway control or emergent cardioversion. Perform Although blood pressure measurements in AF can vary
a brief, focused history and physical examination simulta- due to the irregularity of the pulse,27 this has minimal
neously with the initial interventions. Place patients on a clinical significance in the ED. It is important to note that a
cardiac monitor, apply supplemental oxygen as needed, “normal” systolic blood pressure reading of 100 can be
and establish IV access. Once the patient is stabilized, a falsely reassuring. Patients who are chronically hypertensive
more detailed history and focused physical examination may be in shock at this pressure. Mental status is a better
are indicated. indication of adequate cerebral blood flow.
Look to the respiratory rate and pulse oximetry reading
History for clues to airway or respiratory compromise (perhaps due
Some of the most important historical questions when to PE, CHF, or pneumonia).
evaluating a patient with AF are summarized in Table 2. Fever may indicate sepsis, endocarditis, PE, or thyroid
Begin with questions that will determine stability (or lack storm. In patients with very rapid ventricular response,
thereof), such as chest pain, dyspnea, and syncope. Some consider obtaining a rectal temperature, especially in the
patients may be relatively asymptomatic. elderly. In one recent study, rectal thermometry detected a
Palpitation is the most common presenting symptom fever in 15% of geriatric patients who were afebrile orally.28
associated with AF,8 although chest pain, dyspnea, fatigue, Patient stability can usually be determined by vital
and lightheadedness are common. The character of the chest signs, pulse oximetry, and a brief mental status exam. If the
pain (such as crushing, radiating to the left shoulder, etc.) patient is stable, perform a head-to-toe examination.
may provide clues to ischemia. Pleuritic chest pain can be Perform a good neurological examination, as patients in AF
seen with PE or pericarditis. Syncope with AF is an uncom- can present with stroke as the first indication of AF.16 Subtle
mon presentation that usually indicates significant disease findings, such as tremor, eyelid retraction, hyperhidrosis,
such as sinus node dysfunction, valvular aortic stenosis, hyperpigmentation, vitiligo, or alopecia, suggest underlying
hypertrophic cardiomyopathy, cerebrovascular disease, or autoimmune or thyroid dysfunction. During examination of
an accessory pathway. the neck, pay special attention to the thyroid gland and neck
Establish whether the patient has experienced AF in the veins. Palpate the thyroid for tenderness and/or nodularity.
past. Is this episode new in onset, a recurrence of previous Patients with AF exhibit a loss of alpha waves in the jugular
AF, or chronic in nature? Has the patient ever required venous pulse and demonstrate variable force in the pulsa-
cardioversion? Ask about medications. Does the patient tions of the carotid arteries. Auscultate the heart for rate,
report medications for an “irregular heart,” like digoxin, rhythm, murmurs, and additional heart sounds. In AF, the
amiodarone, or warfarin (Coumadin)? Other medications first heart sound usually varies in intensity. Murmurs
such as thyroid supplements or theophylline suggest the suggest the presence of valvular heart disease. Look and
need to measure drug levels. The acuity of onset may guide listen for signs of heart failure, including jugular venous
therapy as well as disposition. However, Holter monitor distention, rales, wheezes, and pedal edema.
data suggest that the time of symptom onset may be
“Sometimes the heart sees what is invisible to the eye.”
unknown or unreliable.15,25,26
—H. Jackson Brown, Jr.
Determine risk factors for coronary artery disease and
PE. Ask about past medical history, drug and alcohol use, Diagnostic Tests
and medication use, particularly in reference to previous Most patients with NOAF will require some testing in
cardiovascular, pulmonary, and thyroid disorders. A history the ED.29 It is important to note that some diagnostic
of rheumatic fever or IV drug abuse provides clues to recommendations are arbitrary, since there are few, if any,
underlying valvular disease. Ask if the patient has a known prospective studies evaluating their utility. It is almost
pre-excitation syndrome (“I have W-P-something, Doc.”). axiomatic for an ECG and a chest x-ray to be ordered on all
The review of systems should address any metabolic patients with new-onset or uncontrolled AF. Other tests
disturbances or neurological symptoms, such as thyrotoxi-
cosis or acute neurologic events. Ask specific “thyroid
Table 2. High-Yield Historical Questions.
questions” about weight loss or gain, changes in menses,
change in hair loss or texture, and nervousness or behav- Have you had atrial • palpitations?
ioral changes. fibrillation before, and if so: • weakness?
• for how long? • lightheadedness?
• what are you treated • confusion?
Physical Examination
with?
The vital signs are crucial in the assessment of initial patient
• are you on digoxin or Do you have a past medical
stability. Palpation of an irregularly irregular pulse may be warfarin? history of:
the first clue to AF, while AFl usually presents with a • are you compliant? • cardiac, pulmonary, or
regular tachycardia. Consider evaluating AF patients with thyroid disease?
very high heart rates (>160 bpm) for thyrotoxicosis, CHF, Do you have associated • coronary artery disease,
PE, fever, dehydration, or acute blood loss. Conversely, symptoms, such as: or risk factors for
those with inappropriately low untreated heart rates with • chest pain? pulmonary embolism?
NOAF may have underlying conduction abnormalities or • dyspnea? • drug and alcohol use?
perhaps take medications that mask tachycardia. • syncope? • rheumatic fever?

Emergency Medicine Practice 4 www.empractice.net • August 2002


should be based on patient presentation and risk factors for be a diagnostic dilemma. True irregularity typically rules
other underlying disease. out ventricular tachycardia and leaves a diagnosis of a
supraventricular tachycardia with aberrant conduction.
Electrocardiogram The most common causes are AF/multifocal atrial tachycar-
An ECG is the gold standard for the diagnosis of AF/AFl. dia with underlying bundle branch block and/or Wolff-
Obtain an ECG on anyone with suspected or proven AF/ Parkinson-White (WPW) syndrome. (See Figure 3 and
AFl to establish the diagnosis, rule out other dysrhythmias, Figure 4 on page 6.) The absence of P waves establishes
evaluate for ischemic changes, and look for signs of the diagnosis of AF. In this setting, ruling out pre-excitation
previous cardiac disease (e.g., prior MI, bundle branch syndrome (WPW) is crucial. Patients with WPW who are
block, pre-excitation, and left ventricular hypertrophy given AV nodal blocking agents can suffer ventricular fibrillation
[LVH]). Furthermore, an evaluation of the RR, QRS, and QT arrest.31 Suspect pre-excitation syndrome in younger
intervals is important when considering the use of certain patients with AF and those with ECG findings that
medications, particularly antiarrhythmics. suggest pre-excitation. These findings include very fast
Several rules can help establish the correct diagnosis on rates (RR > 250), presence of a delta wave, and wide or
the ECG. (See Table 3.) First, the RR intervals are almost bizarre QRS complexes.
always irregularly irregular during AF. (See Figure 1 and The appearance of atrial flutter on ECG is more
Figure 2.) When the ventricular rate is very slow or very fast organized than AF. (See Figure 5 and Figure 6 on page 6.)
during AF, the RR intervals can appear regular on a quick Many patients have a sawtooth pattern of regular atrial
visual inspection; thus, careful scrutiny of the ECG is activation called flutter waves, particularly visible in leads
important. Calipers are helpful. Doubling the paper speed II, III, and aVF. Untreated, the atrial rate typically ranges
on the monitor or ECG (double-speed strip) to 50 mm/sec from 200-300 bpm with flutter waves inverted on the ECG
can also demonstrate irregularity in very fast rates. In one in leads II, III, aVF, and upright in lead V1. The wave
study of 45 difficult, narrow complex tachycardias (heart activation in the right atrium can occasionally be reversed,
rate range, 150-250 bpm), the correct diagnosis improved resulting in flutter waves that are upright in leads II, III,
from 63% using the standard 25 mm/sec ECG to 71% when aVF, and inverted in lead V1. Two-to-one AV block is
the 50 mm/sec speed was used.30 Also, the absence of P common, producing a ventricular rate of 150 bpm that can
waves helps exclude other supraventricular arrhythmias, be mistaken for another supraventricular tachycardia due to
such as multifocal atrial tachycardia, that can also appear a regular-appearing ventricular response. AFl can coexist
irregular. “Fibrillatory waves” are not always visualized in with AF and can be present on the same ECG.
all patients with AF; conversely, baseline noise or artifact on There are some other “tricks” to picking up AFl on the
the ECG can sometimes simulate these waves in patients ECG that involve modified chest leads, such as the Lewis
who are not in AF. lead and the modified chest lead 1 (MCL1). To monitor the
An irregular tachycardia with wide QRS complexes can Lewis lead, place the right arm electrode is over the right
side of the sternum at the second intercostal space and the
Table 3. ECG Diagnosis Of Atrial Fibrillation. left arm electrode over the fourth intercostal space (again,
on the right side of the sternum). The tracing is then
• Irregularly irregular RR intervals
• Absence of P waves recorded on lead I.32 This lead, which is often perpendicular
• Irregular fibrillatory wave forms (best seen in V1)
Figure 2. Atrial fibrillation with pre-existing LBBB in a
Figure 1. Atrial fibrillation with rapid ventricular 60-year-old woman with hypertension.
response in a 76-year-old man with breathlessness.

Sometimes this can be confused with ventricular tachycardia,


but closer inspection can identify the irregularity. The
Note the irregularly irregular ventricular rhythm. Sometimes
irregularly irregular rhythm suggests AF. Features of typical
on first look, the rhythm may appear regular, but on closer
left bundle branch block include wide QRS (> 120 ms), no
inspection it is clearly irregular.
secondary R wave in lead V1, and no lateral Q waves.
Used with permission from: Jenkins & Gerred,
Used with permission from: Jenkins & Gerred,
www.ecglibrary.com.
www.ecglibrary.com.

August 2002 • www.empractice.net 5 Emergency Medicine Practice


to atrial depolarization, may show the flutter pattern. particularly if there is hypoxia or the history and physical
The use of certain maneuvers to increase the degree of examination suggest cardiac or pulmonary disease.
AV block and slow the QRS rate, such as the Valsalva Evaluate the chest x-ray for the presence of a pulmonary
maneuver, carotid massage, or adenosine, can also help mass, elevated hemidiaphragm, infiltrate, or CHF. The size
determine the rhythm in cases in which it might be unclear. and shape of the cardiac silhouette may hint at cardiomy-
However, some general caution with the use of adenosine in opathy or valvular disease. The classic “water-bottle” heart
wide complex rhythms may be warranted. Cases of with clear lung fields suggests cardiac tamponade. Straight-
deterioration with its use in ventricular tachycardia and ening of the left cardiac border on the PA film can be caused
WPW have been reported, but these cases are rare (the by the dilation of the left atrial appendage that can occur in
pooled literature actually appears to support the overall mitral stenosis/regurgitation. Remember, there can be an
safety of adenosine in this setting).33 However, if the rhythm association between a right middle lobe infiltrate and AF.
is definitely an irregular wide complex tachycardia, Look at the chest x-ray for the signs occasionally seen in PE
adenosine is not effective in terminating or slowing AF and like Hampton’s hump or Westermark’s sign (although
thus is not indicated. infiltrates, atelectasis, or blunting of the costophrenic angle
are more likely presentations of this disease).
Chest X-Ray
A chest x-ray should be obtained in cases of NOAF/AFl, Laboratory Tests
Thyroid Studies
Although it may have an overall low yield,34 a screening
Figure 3. Wolff-Parkinson-White syndrome with atrial
TSH should be obtained in older patients (> 55) with NOAF.
fibrillation in a 47-year-old man with a long history of
The elderly can often have very atypical presentations of
palpitations and, lately, blackouts.
hyperthyroidism or thyrotoxicosis with very few, if any, of

Figure 4. Wolff-Parkinson-White syndrome with atrial


fibrillation in a 23-year-old man with episodes of
palpitations.

Note the irregularly irregular, wide complex tachycardia.


Impulses from the atria are conducted to the ventricles via
either both the AV node and accessory pathway (producing a Used with permission from: Jenkins & Gerred,
broad fusion complex), or just the AV node (producing a www.ecglibrary.com.
narrow complex without a delta wave), or just the accessory
pathway (producing a very broad “pure” delta wave). People
Figure 6. Atrial flutter with 2:1 AV conduction in a 57-
who develop this rhythm and have very short RR intervals are
at higher risk of VF.
year-old lady with palpitations.

Used with permission from: Jenkins & Gerred,


www.ecglibrary.com.

Figure 5. Atrial flutter in a 68-year-old lady on digoxin


complaining of lethargy.

Atrial flutter often produces a characteristic “sawtooth” or The sawtooth waveform of atrial flutter can usually be seen in
“picket-fence” waveform of an intra-atrial re-entry circuit, the inferior leads II, III and aVF if one looks closely. Sometimes
usually at about 300 bpm. This lady was taking rather too the rapid atrial rate can be seen in V1. Suspect atrial flutter with
much digoxin and has a very slow ventricular response. 2:1 block when you see a rate of about 150 bpm.
Used with permission from: Jenkins & Gerred, Used with permission from: Jenkins & Gerred,
www.ecglibrary.com. www.ecglibrary.com.

Emergency Medicine Practice 6 www.empractice.net • August 2002


the other classic signs and symptoms (like buggy eyes, (ACS), and its development tends to be associated with
sweating, tremors, or diarrhea).35,36 Development of AF in higher 30-day and six-month rates of mortality and stroke.7
patients with hyperthyroidism likely accounts for some of Up to 20% of patients who present with acute MI will
the increased mortality from cardiovascular and cerebrovas- develop AF in close proximity to the acute MI.7,41 The
cular disease in these patients.37 Consider obtaining a TSH incidence of MI in patients with AF on presentation to the
in younger patients with signs of hyperthyroidism, espe- ED may be as high as 15% or higher.42
cially those with high heart rates, or when the ventricular Despite the fact that AF is a common complication of
rate is particularly difficult to control. AMI, there are some reasonable—albeit mostly retrospective—
data to suggest that not all patients with NOAF require a
Complete Blood Count routine “rule out” set of cardiac enzymes as part of their
A routine CBC is a low-yield test in most cases of NOAF; diagnostic evaluation. Enzymes should be reserved for
however, the presence of anemia may be an important patients suspected of being at risk for having ACS.42-45 Based
finding. A hemoglobin should be performed when there are on the current data, at-risk patients would include those
physical signs of anemia or a history of recent blood loss. A with ECG changes suspicious for ischemia or underlying
WBC may be elevated as a nonspecific stress reaction or can heart disease (prior MI, LVH, and BBB); those with signifi-
suggest infection but has limited overall utility. A platelet cant risk factors for CAD (patients older than 65, smokers,
count should be obtained in patients at risk for thrombocy- or those with diabetes mellitus, hypertension, high choles-
topenia when anticoagulation is being considered. terol, or previous CAD); and those with symptoms that
otherwise would be considered for possible angina (chest
Chemistries pain, pressure, significant dyspnea, CHF, etc.).42-46 Other-
While they are often ordered routinely, serum chemistries wise, patients with NOAF may not require routine “rule
rarely affect the management of NOAF. Order them if the out” enzymes according to the available data. Still, caution
patient is taking diuretics or if there are other reasons for an is advised until further research is performed. (For a more
electrolyte abnormality. It is important to identify hypokale- detailed discussion on this topic, see the “Disposition”
mia when considering the use of certain antiarrhythmics, as section later in this article.)
this combination is associated with an increased incidence
of torsades de pointes.38,39 Remember that serum magne- Tests For Pulmonary Embolism
sium levels do not correlate well with total body magne- Medical textbooks sometimes suggest that patients with
sium and are of little value. Check a BUN/Cr in patients on NOAF must be admitted to the hospital to rule out PE.
diuretics and in those with a history of dehydration, renal However, studies (mostly retrospective) imply that patients
insufficiency, or signs of fluid overload. with NOAF who otherwise are not suspected of having PE
are unlikely to have the disease and do not require additional
Drug Levels testing.45 It is advisable to exclude this diagnosis if the
Obtain digoxin levels on patients taking this drug. Noncom- patient has significant risk factors for PE or suggestive
pliance with digoxin often results in a rapid ventricular clinical findings.
response in those with chronic AF. Although many
dysrhythmias can be associated with digoxin toxicity Echocardiography
(paroxysmal atrial tachycardia with block, bidirectional Echocardiography (echo) can be a valuable tool to evaluate
ventricular tachycardia, accelerated junctional rhythm), the cardiac abnormalities, and it may have an occasional role in
finding of AF with a slow ventricular response should the acute management of NOAF in the ED. Consider it
prompt its consideration. Digoxin toxicity (but not digoxin (when available) in an unstable patient with AF. Echo can
use) is a relative contraindication to electrical demonstrate signs of right heart strain that might indicate a
cardioversion/defibrillation, as case reports of asystole after diagnosis of PE.47 Echo can also help evaluate left ventricu-
such attempts to treat tachydysrhythmias in this setting lar function and the presence of valvular or pericardial
have occurred.40 pathology. While most patients with NOAF will get an echo
Theophylline use or toxicity can cause AF; obtaining a at some point, this can often be done as part of the inpatient
level in patients on this drug is indicated.9 workup, or possibly on an outpatient basis. Routine use of
echo in the ED is not recommended at this time.
Coagulation Studies There is some emerging literature to suggest that
Signs and symptoms of coagulation abnormalities can transesophageal echo (TEE) may help guide acute conver-
usually be detected on the history and physical examina- sion of patients with AF of acute or unknown duration,
tion. Their presence should prompt an evaluation of the although this is a controversial topic.1,48-54 It is important to
coagulation studies and a platelet count. Obtain a protime note that a transthoracic echo is inadequate for the exclusion
(PT) and international normalized ratio (INR) if the patient of clot, as it cannot visualize the left atrial appendage very
is on warfarin and consider a baseline partial thromboplas- well.50,52,55 If patients have AF of a short duration (<48 hours)
tin time (PTT) if the patient will be placed on heparin. and no clot is present on TEE, some data support early
medical or electrical cardioversion.
Cardiac Markers Echo might also help stratify who might be successfully
AF is a common complication of acute coronary syndrome converted and remain in normal sinus rhythm. Some data

August 2002 • www.empractice.net 7 Emergency Medicine Practice


suggest that left atrial size assessed by echo may predict the Pad Location
outcome of cardioversion and subsequent maintenance of Successful cardioversion requires defibrillation of a critical
sinus rhythm. However, not all studies support this.56,57 mass of the atria. To achieve this goal, a variety of anatomic
pad placements can be used. The apex-anterior, apex-
Treatment posterior, and the anterior-posterior are all effective.56,58
Anterior-posterior placement is possibly more useful than
Determining the stability of a patient is the first task when the anterior-apex method,59,60 although there is controversy
considering treatment of AF/AFl, and this is not as black- regarding this subject.58 Interestingly, some patients who
and-white as texts often imply. The ACLS guidelines do not respond to one method occasionally respond to
suggest that signs and symptoms of instability with AF another method.56
include “chest pain, shortness of breath, dyspnea on
exertion, [and] altered mental status.” The guidelines Technique
recommend electrical cardioversion over medical therapy Regardless of the pad placement, use a synchronized
for the unstable patient.2 In real practice, stability is not delivery to minimize the small risk of inducing ventricular
always a dichotomous issue, but rather a continuum. Surely, fibrillation. For optimal energy delivery, try to shock in full
the patient who is hypotensive, diaphoretic, and who has expiration (the patient’s, not yours), maintain a good
altered mental status is unstable, but not everyone with a interface between the pads and the patient, and avoid the
little bit of chest pain requires a jolt of electricity. The breast tissue in women. Because firm pressure can decrease
emergency physician must always recognize the truly impedance, there may be some theoretical advantage to
unstable patient in addition to being prepared for possible using the old standard manual pads as opposed to the self-
deterioration of the stable patient. adhesive pads. However, the old paddles require the use of
gel and will not always fit on a chest littered with electrodes.
Unstable Patients In fact, studies have shown similar efficacy with both the
If the patient with rapid AF is unstable, then emergent direct paddles and adhesive pads.56
current cardioversion (DCC) is the treatment of choice. Fortu-
nately, this is a rare event; unfortunately, DCC in this setting Energy Level
may be associated with several problems, most pressing of There are few or no data regarding the optimal starting
which is that it might not work. An underlying illness such energy level to use in unstable patients with AF. However,
as a low ejection fraction,57 thyrotoxicosis, or mitral stenosis some inferences can be drawn from the literature regarding
may make conversion impossible, difficult, and/or only DCC in stable patients. Although the American Heart
short-lived. A dilated left atria and a longer duration of AF Association recommends starting at 100J,2 only 50% or fewer
may decrease conversion rates.56-58 (Factors associated with of stable patients will convert with that energy.57,60,61 For this
failed cardioversion are summarized in Table 4.) reason, starting at a higher energy level seems appropriate
Acute cardioversion is associated with a significant risk in an unstable patient. Data show that roughly 75% of
of thromboembolism during or shortly after the procedure, patients with AF will convert with 200J,60 and current trends
particularly if the AF has been present for more than 48 in the literature suggest this approach.1,59 Others recommend
hours. (For more information regarding this topic, see the using an even higher initial energy (360J), particularly if the
“Anticoagulation” section later in this article.) However, AF duration has been more than 48 hours. This is based on a
emergent DCC should not be withheld in the unstable patient due study by Joglar et al, who showed an initial single shock
to this risk. Even if the patient is successfully cardioverted, he success rate of 14% with 100J, 39% with 200J, and 95% with
or she might not necessarily improve due to lack of me- 360J in patients with AF for more than 48 hours.61 Interest-
chanical contraction of the atrium (“atrial stunning”). ingly, the cumulative energy for successful cardioversion
Therefore, although DCC should be attempted in the was less if 360J was initially used. There was no evidence of
unstable patient, failure or short-lived effects should be myocardial damage as assessed by cardiac troponin I levels
anticipated, and other interventions including rate control in the group receiving the high-energy shocks. Lack of
agents should be considered. Additionally, given the myocardial damage from DCC in AF was also seen in
likelihood of failure and risks of thromboembolism, rate another study using even higher energies.62 Thus, when
control might be the first line of treatment in the unstable choosing an initial energy level for DCC of an unstable patient in
patient with known chronic AF. rapid AF, start with at least 200J, and consider 360J.
Energy levels as high as 720J have been used for
elective conversion of refractory chronic AF;63 however,
Table 4. Factors Associated With Failed Cardioversion. special defibrillators are required to administer those energy
• Underlying illness—congestive heart failure, thyrotoxico- levels (plus generous sedation). In addition, a method of
sis, valvular disease “double shocks” using both the apex-anterior and the
• Dilated left atrium anterior-posterior methods at the same time has been
• Longer duration of atrial fibrillation reported with some efficacy in a small number of patients.64
• Too low energy Pretreatment with ibutilide prior to DCC in stable patients
• Technique will increase shock success and decrease the energy
• Other patient factors required.65 However, this might not be feasible in an

Emergency Medicine Practice 8 www.empractice.net • August 2002


unstable patient, as 1 mg of ibutilide IV is given over 10 goal is ventricular rate control. It is the ventricular rate that
minutes (a possibly lethal interval in an unstable patient) mostly governs symptoms. Rate control will likely improve
prior to DCC.65 the patient’s overall hemodynamic status, resulting in better
ventricular filling, less myocardial oxygen demand, and
Biphasic Defibrillators improved cardiac output. Many options for rate control are
If you have a biphasic defibrillator (which requires you to available, and emergency physicians must understand the
know your particular model), the manufacturer recom- indications, contraindications, and nuances of each of these
mends paddle placement for AF in the anterior-posterior interventions. Sometimes patients with chronic AF will present
position with the anterior pad just right of the sternum and acutely with a rapid ventricular response due to a physiologic
the posterior pad over the left scapula.66 Start with 120J stress, such as fever, dehydration, pain, or anemia. Treatment in
synchronized, biphasic shock. Two studies suggest that a such cases would be acetaminophen, fluids, opioid analgesics, and
rectilinear biphasic waveform shock in AF has better success blood transfusion, respectively—not cardioactive drugs.
with less energy requirement compared to monophasic The most common choices for rate control agents can
waveform shock.66,67 be found in Table 5. Calcium-channel blockers, ß-blockers,
and digoxin are not conversion agents and are not intended
Atrial Flutter for such purposes. While 25%-60% of patients with NOAF
As in the case of an unstable patient with AF, DCC is also may spontaneously convert while on these agents, patients
the initial treatment of choice in the unstable patient with who receive no intervention or placebo demonstrate similar
AFl. Approximately 90%-100% of stable patients with AFl conversion rates.70-73 Amiodarone possesses antiarrhythmic
respond initially to DC cardioversion;24,68 however, less in properties in addition to its rate control effects. When
known about success rates in unstable patients. Stable choosing a rate control agent, strongly consider whether or not the
patients with AFl often require less energy to convert than patient could have an accessory pathway; the use of an AV nodal
those patients with AF,58,69 and the current ACLS guidelines blocking agent would be contraindicated in that situation. (See
suggest starting with 50J.2 However, there are scarce data in the “Special Circumstances” section later in this article.)
the current literature regarding the optimal initial energy
choice for DCC of AFl in the unstable patient. There is Cardioversion
reasonable evidence to suggest that in a certain percentage DCC is the quickest way to obtain rate control, even in a
of patients with AFl, starting with lower energy levels will stable patient. However, it requires procedural sedation,
induce AF (an objectionable event in an already unstable with all of its accompanying risks. Acute DCC is also
patient).58,68,69 This would suggest that 200J (or more) may be associated with some potential complications, particularly
preferable to the recommended 50J.68,69 embolic events and cardiac arrhythmias, which should be
minimized prior to “elective” cardioversion of a stable
Rate Control patient (see below). However, in an unstable patient, DCC
may save a life. It has been shown to be safe and efficacious
Once the patient with rapid AF has been stabilized, the next in the hands of emergency physicians when managing

Table 5. Rate Control Agents.


Modality Administration
Electrical cardioversion Start with 200J synchronized, monophasic shock or 120J synchronized, biphasic shock*
Calcium-channel blockers
• Diltiazem 0.25 mg/kg IV over two minutes followed by a second bolus of 0.35 mg/kg IV over two minutes
if there is inadequate response at 15 minutes
or 5 mg increments slow IV up to 50 mg followed by drip 5-15 mg/h
• Verapamil 2.5-5.0 mg over 2-3 minutes followed by 5-10 mg in 15-30 minutes if necessary ± drip 5 mg/h
or up to 0.005 mg/kg/min
ß-blockers
• Metoprolol 5 mg IV every 5 minutes up to 15 mg
• Propranolol 1 mg IV every 2 minutes to max of 5-8 mg
• Atenolol 5 mg IV over 5 minutes; repeat after 10 minutes if inadequate response
• Esmolol 0.5 mg/kg IV bolus over 1 minute, followed by a 0.05-0.2 mg/kg/min IV infusion. Repeat cycle and
increase drip if no effect.
Cardiac glycoside
• Digoxin Load 0.5 mg IV and repeat 0.25 mg every 4-6 hours for three doses
Others
• Amiodarone 150 mg IV over 10 minutes followed by infusion of 1 g over six hours. May repeat bolus if needed.
• Magnesium 2 g bolus over 10-15 minutes followed by 1 g/h infusion
• Clonidine 0.1 mg PO
* With atrial flutter, consider lower starting energy (100J), monophasic to start

August 2002 • www.empractice.net 9 Emergency Medicine Practice


stable23 as well as unstable patients with AF. this debate. Most of the debate regarding which agent
produces less hypotension stems from animal data and
Calcium-Channel Blockers studies on diseased human heart tissue performed by Bohm
Calcium-channel blockers (CCBs) are the first-line treatment for et al.75,77 They suggested that verapamil caused greater
rate control in rapid AF.74 They are efficacious, act quickly, and reduction in contractility than diltiazem, but these findings
are relatively safe. The two most commonly cited agents have been challenged.76,80,84,88,90
used in the ED are diltiazem and verapamil; much contro- The only “head-to-head” human trial did not settle the
versy exists in the literature as to which agent is better.75-79 debate.78 This randomized, double-blind, crossover trial
They both act on the sinoatrial and AV nodes but do not compared the effects of IV diltiazem and verapamil in 17
prolong the refractoriness of accessory pathways. Both adults with AF or AFl and a baseline heart rate of 120 bpm
agents reduce systemic vascular resistance, and the most or higher. No pretreatment with calcium was used in the
frequent complication is hypotension.80 The risk of hypoten- trial. Both agents showed similar efficacy for heart rate
sion can be minimized by the use of IV calcium as a pretreatment control at two minutes; however, three of the 17 patients
prior to giving these drugs, or as a rescue agent if hypotension initially entered into the study developed symptomatic
occurs.81-83 CCBs are effective in controlling ventricular hypotension after verapamil, which prompted withdrawal
response during exercise in chronic AF.84 from the study. Hypotension was considered life-threaten-
ing in one patient who was found to have an ejection
Pretreatment With Calcium fraction of 20%. This study has led many to declare that
If a patient develops hypotension in the setting of receiving clearly diltiazem is better than verapamil because of the
a CCB, it can often be reversed or attenuated with IV hypotension. However, the trial was small, and the dosing
calcium. Life-threatening tetany has also been reported after protocol of the study still leaves this question open to
IV diltiazem that responded to IV calcium chloride.85 debate. In the patient who has a borderline blood pressure,
Emergency physicians treating AF/AFl patients with CCBs it may be prudent to use diltiazem. However, the choice of
should have quick access to IV calcium. drug may be less important than how much and how
Alternatively, calcium as a pretreatment should be quickly it is given. Titrate CCBs over several minutes.
considered in patients with marginal blood pressure prior to Another reason that diltiazem is commonly quoted as
receiving a CCB.81-83 Calcium has been shown to be effica- being better than verapamil is that diltiazem can be
cious in preventing the negative inotropic effects of the followed by a drip. A diltiazem bolus followed by a drip did
CCBs but does not affect the drugs’ chronotropic effects. show better efficacy compared with a diltiazem bolus
Side effects of calcium pretreatment are rare and mild, followed by placebo over 24 hours, which is not surprising
including flushing and a feeling of “generalized heat” given the half-life of the drug. However, a verapamil drip
lasting 2-3 minutes.81 While the optimal dose for pretreat- has been described and is effective.95 (However, since not
ment is not known, it seems reasonable to use 1-3 cc of many people are familiar with it, you might get some blank
calcium chloride or 5-10 cc of calcium gluconate. stares if you order one.) Some evidence suggests that
patients may be quickly switched to oral forms of the CCBs
Verapamil vs. Diltiazem to maintain adequate rate control, possibly avoiding the
Comparison trials of CCBs (mainly verapamil and issue of a drip altogether.96 While verapamil is cheaper than
diltiazem) with placebo in rapid AF show that the CCBs are diltiazem by about $10, this difference is unlikely to start a
more than 90% successful in reducing heart rate.80,84,86-89 This run on the bank.
holds true for patients with AFl as well.80,87 CCBs quickly
control ventricular response—often in less than five Contraindications/Cautions
minutes—when given intravenously.87,90 CCBs are more Kuhn and Schriger published a retrospective review of
effective for acute rate control than digoxin.90-92 In a compari- patients who received verapamil despite a contraindication,
son study, Schreck et al showed a statistically significant and they reported a higher incidence of adverse outcomes
reduction in heart rate for diltiazem-treated patients at five and drug failures in that group compared with those who
minutes vs. 180 minutes in digoxin-treated patients.90 The did not have a contraindication.97 CCBs are contraindicated
combination of IV diltiazem with IV digoxin may result in in patients with an accessory pathway, because these
more efficacious ventricular rate control with fewer patients may develop a rapid ventricular response and
fluctuations than that of IV diltiazem alone;93 however, some induction of ventricular fibrillation secondary to reflex
feel the addition of digoxin is unnecessary. In critical care sympathetic stimulation of the accessory pathway.31,33
patients, diltiazem slows the heart rate more profoundly Hypotension is the most common significant side effect of
than IV amiodarone, although it produces significantly CCBs.81-83 (See Table 6 on page 11.)
more hypotension.94 Special mention of the use of CCBs in patients with
So which agent should we use—diltiazem or signs of CHF is worthwhile. CCB use in patients with CHF
verapamil? Advocates of diltiazem state that it has less risk is controversial according to some authors, but the literature
of hypotension and that patients can be maintained on a suggests overall safety and efficacy.84,98-100 Rapid AF with
drip after the initial bolus. Verapamil users state that neither CHF is not necessarily a contraindication to CCB use; in fact,
agent has been shown to be definitively better and that it may be part of the appropriate treatment, particularly in
verapamil is cheaper. There is some truth to both sides of relatively stable patients, and perhaps in unstable patients in

Emergency Medicine Practice 10 www.empractice.net • August 2002


whom cardioversion fails. (Remember that the moribund, rapidly after IV administration, and adequate rate control
severely dyspneic patient with acute CHF and rapid AF can often be established within two minutes of drug
should be considered unstable and likely have attempts at administration.101-103 There is likely some benefit to using
cardioversion prior to medical therapy.) these agents as the first line in clinical scenarios associated
In the setting of rapid AF with signs of CHF, the use of with high adrenergic tone, such as thyrotoxicosis, acute
CCBs may improve cardiac function by slowing ventricular hypertensive crisis, and MI. Particularly in MI patients,
rate, decreasing oxygen demand and increasing diastolic ß-blockers decrease both short-term and long-term morbid-
filling time. Several small studies in patients with Class 3 or ity and mortality.
4 heart failure have established CCB relative safety in this Contraindications to ß-blockers are very similar to
setting.84,99,100 In a study by Goldenberg et al of 37 patients CCBs, with the same risks in accessory pathways, VT, and
with rapid AF and moderate-to-severe CHF, 97% responded high-degree blocks. In addition, ß-blockers are also contrain-
well to one or two doses of diltiazem followed by a drip. dicated in patients with active bronchospasm or severe
Hypotension was the most common adverse event (11%), asthma, although the ß1-selective agents may have fewer
but in all cases, patients responded with nonpharmacologic problems associated with their use. ß-blockers do have
treatment. No patient was reported to have an exacerbation negative inotropic effects and thus must be used with
of the CHF due to the diltiazem bolus or the drip.84 caution in patients with LV dysfunction. The most fre-
Heywood et al showed similar results with eight out of nine quently reported significant side effect is hypotension.101-103
patients with clinical signs of CHF and rapid AF responding Esmolol, a ß1-selective agent, has a rapid onset and an
to diltiazem without clinical and objective signs of worsen- ultra-short half-life (7-9 minutes), which has some benefits
ing.100 However, in the trial by Phillips et al, the three in patients with rapid AF. Plattia et al found IV esmolol to
patients who received verapamil and became hypotensive have similar efficacy to verapamil for rate control in AF.101
had moderate-to-severe LV dysfunction, leading the authors Hypotension was reported in both groups, but the authors
to recommend that IV verapamil be administered with preferred esmolol because of its short half-life and noted
caution to control the ventricular response in patients with that the infusion could be stopped if the patient suffered
AF or AFl and concomitant LV dysfunction.78 side effects. Treatment with esmolol is somewhat cumber-
some, however, as it requires repeated bolus dosing and
Dosing multiple infusion steps. It is also expensive compared with
The recommended manufacturer dosing for diltiazem is other agents. Alternatives to esmolol include IV propranolol
0.25 mg/kg IV over two minutes followed by a second (a nonselective ß-blocker) and metoprolol (a ß1-selective
bolus of 0.35 mg/kg IV over two minutes if there is inad- agent). Both of these agents have the disadvantage of longer
equate response at 15 minutes.79,80 Due to the occurrence of half-lives, but the advantages of much lower cost and ease
hypotension, some authors have recommended starting at of administration. Propranolol is a non-cardioselective ß-
even lower doses, perhaps 0.15 mg/kg IV,79 although blocker and must be used cautiously in patients with a
published data with this dose are limited. Others have history of bronchospasm.
recommended giving 5 mg increments IV slowly up to a
maximum of 50 mg. The initial dose(s) can be followed by a Digoxin
drip of 5-15 mg/h for maintenance therapy.80 Verapamil Digoxin has been used to treat AF for more than two
dosing should be 2.5-5.0 mg over 2-3 minutes followed by 5- centuries. Until recent years, digoxin was almost mandatory
10 mg in 15-30 minutes if necessary. As discussed previ- in a patient with NOAF. It has fallen out of favor due to its
ously, it can be followed by a drip as well, typically starting lack of efficacy compared to other agents, its hazardous
at 5 mg/h or up to 0.005 mg/kg/min.79 With either drug, side-effect profile, and the arrival of newer agents. It is now
start with lower dosages, particularly in patients with considered second (or third) line for rate control.104 Al-
underlying CHF or borderline blood pressure, and perhaps though digoxin has some direct influence on the atria and
the elderly. AV node, it affects the autonomic nervous system as well. In
AF, the drug exerts its main electrophysiologic effect
ß-Blockers indirectly, by modulating vagal tone. In the resting patient
A number of different ß-blockers are available (see Table 5 on digoxin, vagal influences on the AV node and atrium are
on page 9) that are highly effective in controlling the rapid enhanced, and, as a result, the ventricular rate is slowed.
ventricular rate associated with NOAF. ß-blockers work by As digoxin’s predominant rate control effects are due to
decreasing the conduction through the AV node. They act increased vagal tone, it loses its effectiveness as a rate
control agent when a patient is active or if there is increased
Table 6. Contraindications To The Use Of Calcium- autonomic sympathetic stimulation. This loss of vagal
Channel Blockers. stimulation in the setting of increased sympathetic tone
would also make digoxin a poor choice in patients with
Second- or third- degree heart block or sick sinus syndrome
rapid AF associated with fever, thyrotoxicosis, hypoxia, or
Severe hypotension
Cardiogenic shock acute blood loss. Some authors suggest that the physician
AF/AFl with a known or presumed accessory bypass tract should ask the question, “If this patient were in sinus
Concurrent use or in close proximity to IV ß-blockers rhythm, would he or she have a sinus tachycardia?”104 If the
Ventricular tachycardia answer is yes, then digoxin alone is unlikely to control the

August 2002 • www.empractice.net 11 Emergency Medicine Practice


ventricular rate, and other agents should be considered. reported,108,113 as have exacerbations of bradyarrhythmias.114
Digoxin does have some modest positive inotropic effects However, amiodarone appears to have few or no negative
likely due to effects on calcium, and it might still have a role inotropic effects and appears relatively safe in patients with
in the management of AF patients with CHF or low cardiac CHF and LV dysfunction.107 Superficial thrombophlebitis
output. One study showed digoxin’s use with esmolol may can occur when the drug is administered through a periph-
have helped prevent hypotension, perhaps due to its eral vein.108,111,113 Because it works on accessory pathways as
positive inotropic effects.102 Another study showed some well as on AV nodal conduction, amiodarone may be useful
possible benefit using a combination of diltiazem and in WPW and other pre-excitation syndromes. However, case
digoxin over digoxin alone in 52 patients with rapid AF.93 reports of VF have been reported with its IV administration
Some of the other limitations of digoxin in rapid AF are in WPW patients,115 and the drug’s peculiar combination of
its slow onset time and some potential toxicity, particularly different electrophysiologic actions might cause differing
with long-term use. It takes 30-180 minutes to slow the effects when given intravenously vs. orally. Disadvantages
ventricular rate significantly,70,71,105 compared with CCBs and include cost, variable effectiveness, and side effects that can
ß-blockers, which often work within minutes.90,101 Digoxin be significant if given long term. The drug is generally well-
blocks the AV node but not accessory pathways and thus is tolerated in the short term.
contraindicated in patients with WPW. It does not convert The current literature does not support the routine use
AF, and there is no evidence that it has primary antiarrhyth- of amiodarone in AF in the ED for treatment of most cases of
mic actions. This has been shown consistently in three AF/AFl. An emergency physician might consider its use in
randomized clinical trials in which digoxin, in the absence the patient with an irregular wide complex tachycardia.
of heart failure, was no better at conversion than pla-
cebo.70,71,105 Furthermore, based on animal data, digoxin may Magnesium
increase the chance of a recurrence of AF after restoration of Magnesium, which some advocates declare a “wonder
sinus rhythm by attenuating the recovery from electrical drug,” is useful in the management of ventricular
remodeling of the atria.106 A small upside of digoxin is that it dysrhythmias, and it has been studied in atrial arrhythmias
costs about as much as water (unless you buy the ultra- as well. Magnesium likely exerts its effects in AF by
pure, spring water in designer bottles). In summary, digoxin decreasing conduction within the AV node. Several small
is a drug that will likely continue to fall in and out of favor. studies have suggested its effectiveness as a rate control
However, except maybe in rare circumstances (e.g., CHF), it agent in rapid AF;116-119 however, its use is most often
is a drug that has been replaced with better, safer, and more recommended as adjunctive therapy, commonly with
efficacious agents. digoxin. Both Brodsky et al120 and Hays et al116 have reported
synergistic effects of digoxin with magnesium. Usually
Amiodarone magnesium is given as a bolus of 2 g over 10-20 minutes,
Although amiodarone has been used to control AF, it sometimes followed by an infusion (studies use different
is not approved in North America for any supraventricular regimens). Rate control is relatively rapid, reported to be
arrhythmia. Despite this, IV amiodarone has been evaluated within five minutes.116 It is well-tolerated, with mild
in the treatment of AF for its rate control properties and flushing, warmth, and tingling accounting for the majority
antiarrhythmic effects. Amiodarone is classified as a Class of side effects, which are likely related to the rate of the
III antiarrhythmic agent but also does have effects similar infusion.116 Occasionally, with rapid infusion or large doses
to Class I, II, and IV agents. Most studies with amiodarone (8-10 g), respiratory muscle fatigue, hypotension, and
show that it slows ventricular rate in rapid AF typically cardiac pauses can be seen.120 The drug can accumulate
with significant rate control effects within 20 minutes of a rapidly in patients with renal failure and should generally
bolus administration.72,94,107,108 Conversion rates are highly be avoided in these patients. In addition to its good safety
variable (4%-100%) depending on study characteristics. profile, magnesium has the benefits of being very inexpen-
Amiodarone is slower in onset and less effective than sive, readily available, and easily administered.
diltiazem for rate control but may have less associated Studies with magnesium show that it controls rate
hypotension in critically ill patients.94 Several dosing when given alone116 and can convert 50%-60% of patients
regimens of amiodarone have been used, and no large with recent-onset atrial tachyarrhythmias.117,119,120 A study
trial comparing the efficacy and safety of the different comparing magnesium with amiodarone in critical care
dosages in rapid AF has been performed. If you choose patients with atrial tachyarrhythmias (most of which were
to use amiodarone for rapid AF, give an initial loading AF) found that magnesium controlled ventricular rate just
bolus followed by an infusion.94 as well as amiodarone.118 In addition, at 24 hours, the
Despite causing a prolongation of the QT, amiodarone magnesium group had significantly more patients convert
appears to have a relatively low frequency of pro-arrhyth- to sinus rhythm compared with the amiodarone group.
mic effects,109 and the incidence of torsades de pointes is less Similarly, in a study of 46 patients who presented with a
than 1% according to two evidence-based reviews.107,110 prolonged episode of paroxysmal AF, more patients (57%)
Amiodarone in the acute setting can result in hypotension, converted with magnesium at six hours compared to
which is usually transient and well-tolerated. Hypotension diltiazem (22%).119 Although largely unproven, magnesium
generally responds to IV fluids and stopping the drug.72,110-112 therapy might be particularly useful in alcoholics, who
Occasional, seemingly rare episodes of LV failure have been typically suffer from total body magnesium depletion.121 In

Emergency Medicine Practice 12 www.empractice.net • August 2002


summary, magnesium is efficacious as a rate control agent anticoagulation was presumed safe. One study confirmed
in rapid AF, and it might have some antiarrhythmic effects that the occurrence of embolic events in patients without
compared to standard agents. prior anticoagulation was very low, with either spontaneous
or active conversion within the first 48 hours.123 The rate of
Anticoagulation embolic events in this study (0.8%, with a confidence
interval up to as high as 2.4%) was very similar to the
Traditionally, in the ED, the patient with NOAF has been reported incidence of embolism after “standard therapy” of
stabilized, rate controlled, and admitted for further treat- anticoagulation for 3-4 weeks followed by conversion.
ment, workup, and testing. Recently, this dogma has been Within the past few years, the safety of acute conver-
challenged in the literature and in practice. Some suggest sion of AF with an onset time of less than 48 hours has been
that certain patients with NOAF can be converted (either questioned. Patients can be wrong or uncertain regarding
chemically or electrically) in the ED and sent home. This the duration of symptoms.15,25,26 The use of TEE has revealed
practice is somewhat controversial.43-45 Whether patients that up to 13% of patients with onset time of AF of less than
with AF/AFl are admitted, converted acutely, or dis- 72 hours have a clot present in the atrium, which appears to
charged, thromboembolism is a concern. Some data do place them at a higher risk of embolization.124 For this and
suggest who is at most risk and how anticoagulation other reasons (such as atrial stunning), some authorities
might prevent stroke or other complications. However, recommend that most NOAF patients should be anticoagu-
anticoagulation is not without its own risks. Roughly lated (unless contraindicated) prior to conversion regardless
4%-18% of patients with conventional anticoagulation of the onset time, and the anticoagulation should continue
therapy for seven weeks surrounding conversion will after conversion for a period of 3-4 weeks.48,125
have a minor bleeding episode, while 1.5%-2.0% will have
a major event.48,54 Role Of Transesophageal Echocardiography
AF, and to a lesser extent AFl, are associated with an TEE may help determine the duration of anticoagulation
increased long-term risk of stroke6,16,17 as well as an in- necessary before cardioversion. A large, multicenter,
creased risk of thromboembolism in the post-conversion randomized, prospective clinical trial has compared
period.14 The dysfunctional atria in AF/AFl cause stagnation conventional therapy to the use of TEE with brief prior
of blood flow, which can lead to clot formation and subse- anticoagulation for the conversion of patients with onset of
quent embolization prior to or after conversion.14,122 Conver- AF of more than 48 hours.54 The study showed similar
sion of AF to sinus rhythm results in further transient embolic rates and risk of hemorrhage in conventionally
mechanical dysfunction of the left atrium and left atrial treated patients (anticoagulation for three weeks prior to
appendage, known as “stunning.”122 Recovery of the conversion) and in patients converted after brief anticoagu-
mechanical dysfunction may take up to several weeks, lation and a negative TEE. Patients in the TEE group were
which could explain why patients with a negative TEE prior treated with IV unfractionated heparin (target-activated
to cardioversion can develop post-conversion thromboem- PTT, 1.5-2.5 times the control value), and cardioversion was
bolic events. The overall risk of thromboembolism in all performed within 24 hours. However, this technology is
patients with AF after conversion appears to be as high as readily available in few EDs.
5%-7% without anticoagulation.1,48,123 The overall risk
appears to drop to 0%-1.6% if conversion is preceded by 2-4 Recommendations
weeks of anticoagulation or shorter-term anticoagulation So what should emergency physicians do regarding
and a negative screening TEE.48,54,123 (Circumstances anticoagulation for those patients with AF they see in the
associated with a high risk of thromboembolic events are ED? The answer is not completely straightforward. (See also
summarized in Table 7.) the Clinical Pathway, “Anticoagulation With Atrial Fibrilla-
The point at which a patient with NOAF develops a tion,” on page 17.)
significant risk of clot formation and subsequent emboliza-
tion is controversial. Traditionally, it has been felt that the AF Duration Of 48 Hours Or More
risk of clot formation is very low in the first 48-72 hours of In patients whose duration time in AF is unknown or
the onset of AF, and acute conversion without prior clearly 48 hours or more, the literature supports anticoagu-
lation prior to conversion. This is true even if a TEE will
Table 7. Risk Factors For Thromboembolism With be used in the decision analysis. Whether anticoagulation
Conversion Of Atrial Fibrillation. needs to begin in the ED prior to hospital admission
remains unknown.
Advanced age
Rheumatic mitral valve disease
AF Duration Of Less Than 48 Hours
History of diabetes
With patients whose onset time is known to be less than
Previous stroke or transient ischemic attack
History of hypertension 48 hours, the role of anticoagulation is less clear.
Left ventricular dysfunction Options include:
Prolonged atrial fibirllation 1. Perform blind cardioversion (no echo) in low-risk
Hypercoagulability patients; avoid in high-risk patients (see Table 7).
Presence of clot on TEE 2. Perform a TEE:

August 2002 • www.empractice.net 13 Emergency Medicine Practice


• if negative, cardiovert the patient without admitted developed enzymatic evidence of non-Q-wave MI.
anticoagulation. Risk factors for complications (including MI) were signs of
• if positive, anticoagulate for three weeks with CHF on arrival and lack of conversion to normal sinus
follow-up TEE.53,55 rhythm within six hours. Zimetbaum et al published a
3. Anticoagulate all patients regardless of risk. prospective cohort study in which they reported on 109
patients who underwent a standard “rule-out MI” proto-
A discussion of these options with the patient’s primary col.46 They found ECG changes to be the most important
care physician and/or a cardiologist may be helpful. If discriminator as to which patients were likely to develop
a patient with NOAF is already on anticoagulants (war- MI. Chest pain was a very sensitive, but nonspecific, marker
farin), cardioversion in those with an adequate INR of MI. However, this study shows that those patients with
might be reasonable. NOAF and chest pain (how many don’t have some chest
Anticoagulation does not always require an admission. pain?) require admission to evaluate for ischemia. After
In the past, if the decision to start anticoagulation was made, pooling the data from these studies and another retrospec-
then heparin was started in the ED and the patient was tive study by Mulcahy et al,43 it does appear that some very
admitted. However, a dose of heparin followed by immedi- select patients with NOAF are at low risk for MI and may
ate cardioversion is not warranted in those with AF of not require admission. (See Table 8.) It is important to note
greater than 48 hours’ duration except perhaps in unstable that these criteria have not been prospectively evaluated.
patients.1 (Anticoagulation should not delay cardioversion
for the moribund patient, but it should be started afterward Admission Criteria
if not contraindicated.) Mulcahy et al published a study that specifically looked at
The introduction of low-molecular-weight heparin NOAF patients and tried to address the question of when
(LMWH) creates the opportunity for outpatient anticoagula- admission is medically justified.43 They looked retrospec-
tion of AF/AFl. Two small studies have evaluated LMWH tively at 216 patients admitted to the hospital with NOAF.
in patients with AF and found it to be efficacious at preven- They believed that only 143/216 (66%) of the admissions
tion of embolization in patients with chronic AF.126,127 A TEE- were medically justified based on complications they had or
guided conversion trial with the use of LMWH is currently developed in the hospital. Of the 143 patients who were
ongoing (ACUTE II), and a cost-analysis model suggests ultimately classified as medically justified admissions, 140
this approach could be cost-effective.51 (98%) could have been predicted from criteria fulfilled while
in the ED. They suggested that patients with NOAF as the
“I believe every human has a finite number of heartbeats. sole reason for admission (33% in this study) “can be
I don’t intend to waste mine running around doing exercises.” managed as outpatients if they are hemodynamically stable
—Neil Armstrong and the ventricular rate can be controlled in the ED.”
However, the authors rightfully point out that their study
Disposition was retrospective and should be validated in a prospective
trial. It is interesting to note, however, that two of the three
Traditionally, patients with NOAF were admitted to the patients who would not have met the author’s admission
hospital for further treatment, evaluation, anticoagulation, criteria, but were still admitted, died. In addition, patients
and possible conversion. Admission has also been recom- categorized as not medically justified still spent several days
mended for ruling out serious causes of AF that might cause in the hospital.128
morbidity and mortality, namely MI and PE. Recently, this
routine admission of all patients with NOAF has been Discharge Criteria
challenged in the literature. However, support for discharg- So who are the truly low-risk patients who might go home
ing low-risk patients is largely retrospective and must be safely? Suggested criteria at this time would include:
interpreted with caution. • younger patients (<60);
• patients without significant comorbid disease;
New-Onset Atrial Fibrillation Continued on page 19
And Myocardial Infarction
Several studies have evaluated the usefulness of admitting Table 8. Predictors For Myocardial Infarction
or obtaining enzymes in all patients with NOAF to “rule In New-Onset Atrial Fibrillation. *
out” MI.42-46 In a small retrospective study, Friedman et al
showed that all of the MIs (n=5) in the setting of NOAF ECG changes
could have been predicted on arrival to the ED from a set of Other ECG findings—left ventricular hypertrophy, bundle
branch block, old MI
high-risk criteria. These criteria included left ventricular
Chest pain
hypertrophy, ECG evidence of old MI, cardiac-sounding
Signs of congestive heart failure
chest pain, and cardiac symptoms of less than four hours.44 Hypotension
Friedman et al then prospectively examined a group of Suspicion for MI
patients admitted with NOAF (without obvious evidence of Age over 65
MI on arrival) to identify risk factors associated with in-
*Important to note—these have not been validated in a large,
hospital complications.42 Of note, 15% of the patients prospective fashion.

Emergency Medicine Practice 14 www.empractice.net • August 2002


Clinical Pathway: Managing Unstable Patients
With Atrial Fibrillation
Assess patient stability

Unstable Stable


Anticipate instability
Electrical cardioversion (Class I)


• Monophasic 200J-360J (Class I)
• Biphasic 120J (Class II)
• If atrial flutter, may consider lower energy Go to “Clinical Pathway: Rate Control For Stable Patients
• Anticipate failure With New-Onset Atrial Fibrillation With Rapid Ventricular
Response” on page 16

Success?

Yes No

• Reassess stability • Increase energy level incrementally (Class I)


• Further stabilize*
• Begin workup

• Anticipate
decompensation Success?
• Admit
(Class I) Yes No


• Reassess stability Suspicion for accessory pathway?
• Further stabilize*
• Begin workup Yes No


• Anticipate
decompensation
• Admit • Procainamide (Class II) • Ibutilide 1 mg over 10
(Class I) or minutes followed by
• Ibutilide 1 mg IV over 10 cardioversion (Class II-III)
minutes (Class II-III) • Rate control agents
or • Diltiazem (Class II)
• Amiodarone (Class II-III) • Magnesium (Class II-III)
or • Amiodarone (Class II-
• Further electrical III)
cardioversion • Procainamide (Class II)
• alternate paddle • Further cardioversion
placement (Class II) (Class II)
• alternate paddle
placement

* Consider post-conversion or concomitant anticoagulation when possible and not contraindicated—especially if onset unknown
or greater than 48 hours

The evidence for recommendations is graded using the following scale. For complete definitions, see back page. Class I: Definitely recommended.
Definitive, excellent evidence provides support. Class II: Acceptable and useful. Good evidence provides support. Class III: May be acceptable,
possibly useful. Fair-to-good evidence provides support. Indeterminate: Continuing area of research.

This clinical pathway is intended to supplement, rather than substitute for, professional judgment and may be changed depending upon a
patient’s individual needs. Failure to comply with this pathway does not represent a breach of the standard of care.
Copyright ©2002 EB Practice, LLC. 1-800-249-5770. No part of this publication may be reproduced in any format
without written consent of EB Practice, LLC.

August 2002 • www.empractice.net 15 Emergency Medicine Practice


Clinical Pathway: Rate Control For Stable Patients
With New-Onset Atrial Fibrillation With Rapid Ventricular Response
Anticipate instability


Suspicion for accessory pathway?

Yes No


Indication for anticoagulation? Suspicion for MI/thyrotoxicosis?

Yes No Yes No


Contraindication to Contraindication
Anticoagulate (Class I) ➤ sedation? to ß-blocker?

Yes No Yes No


• Procainamide (Class II) • Electrical cardioversion Significant? Use ß-blocker for rate
• Amiodarone (Class III) (Class I-II) control (Class I-II)
Yes No
or


• Procainamide (Class II)
Congestive heart
or Avoid ß-blocker Esmolol
failure or
• Amiodarone (Class III) (Class I) (Class II)
borderline blood
pressure?

Yes No


Contraindication to calcium-channel blockers? Contraindication to calcium-channel blockers?

Yes No Yes No


Consider: Pretreat with calcium— Consider: Consider pretreatment
• Esmolol (Class II-III) 5 cc calcium gluconate • Esmolol (Class II-III) with calcium (Class II)
• Cardioversion (Class II) slow IV (Class I-II) • Cardioversion (Class II)
• Consider anticoagula- • Consider anticoagu-

tion and sedation lation and sedation

issues issues • Verapamil (Class I-II) 2.5


• Magnesium (Class II-III) Diltiazem (Class II-III) • Magnesium (Class mg IV q10-15min until
• Digoxin (in addition to • 5 mg slow IV q5min II-III) rate <120, conversion,
another agent) (Class II) until rate < 120, • Digoxin (in addition hypotension, max 20 mg
• Amiodarone (Class II-III) conversion, or 50 to another agent) or
mg max (Class II) • Diltiazem (Class I-II) 0.25
• Consider drip after • Amiodarone (Class mg/kg IV over 2 min
bolus (Class I-II) II-III) (followed by bolus of 0.35
mg/kg IV over 2 min if
inadequate response at
15 minutes) until rate
The evidence for recommendations is graded using the following scale. For complete definitions, see back page.
<120, conversion,
Class I: Definitely recommended. Definitive, excellent evidence provides support. Class II: Acceptable and useful.
Good evidence provides support. Class III: May be acceptable, possibly useful. Fair-to-good evidence provides hypotension, max 50 mg
support. Indeterminate: Continuing area of research.

This clinical pathway is intended to supplement, rather than substitute for, professional judgment and Consider adjunct treatment
may be changed depending upon a patient’s individual needs. Failure to comply with this pathway • Magnesium (Class II)
does not represent a breach of the standard of care. • Digoxin (Class II)
Copyright ©2002 EB Practice, LLC. 1-800-249-5770. No part of this publication • Avoid ß-blockers (Class
may be reproduced in any format without written consent of EB Practice, LLC. I-II)

Emergency Medicine Practice 16 www.empractice.net • August 2002


Clinical Pathway: Anticoagulation With Atrial Fibrillation
Therapeutic on anticoagulant?

Yes No


Onset time of AF?
Consider for cardioversion
(Class II)
< 48 Unknown or


hours > 48 hours

Risk for thromboembolism?


• Advancing age Ongoing or active bleeding?
• Previous cerebrovascular
accident/transient Yes No


ischemic attack
• Mitral valve disease
• LV dysfunction • Control bleeding • Anticoagulate (Class II)
• History of hypertension, • Reassess risk for • Observe for spontane-
diabetes mellitus anticoagulation ous cardioversion
• Hypercoagulable • Do not convert actively (Class II)
unless unstable or
(Class II-III) • Arrange inpatient or
High Low outpatient conversion

depending on stability
Significant risk of • Consider blind (Class II-III)
bleeding or cardioversion (Class II)
anticoagulation or
contraindicated? • Observe for spontaneous
cardioversion (Class II)

Yes No

• Consider blind cardioversion (Class II) • Strongly consider anticoagulation prior to cardioversion
or (Class II-III)
• TEE (Class II)

(+) clot (-) clot


• Admit and observe Consider cardioversion


(Class indeterminate) (Class II-III)
• Reassess risk of
anticoagulation (Class
indeterminate)

The evidence for recommendations is graded using the following scale. For complete definitions, see back page. Class I: Definitely recommended.
Definitive, excellent evidence provides support. Class II: Acceptable and useful. Good evidence provides support. Class III: May be acceptable,
possibly useful. Fair-to-good evidence provides support. Indeterminate: Continuing area of research.

This clinical pathway is intended to supplement, rather than substitute for, professional judgment and may be changed depending upon a
patient’s individual needs. Failure to comply with this pathway does not represent a breach of the standard of care.
Copyright ©2002 EB Practice, LLC. 1-800-249-5770. No part of this publication may be reproduced in any format
without written consent of EB Practice, LLC.

August 2002 • www.empractice.net 17 Emergency Medicine Practice


Clinical Pathway: ED Cardioversion Of Stable Patients
With Atrial Fibrillation*
• Consider need for anticoagulation (see “Clinical Pathway: Anticoagulation With Atrial Fibrillation“ on page 17)
• Anticipate instability


Contraindication for sedation?

Yes No


Contraindication to medical cardioversion?
• Sedation and analgesia (Class I) followed by:
• hypokalemia
• Direct current cardioversion
• suspected magnesium deficiency
200J monophasic (Class I-II)
• prolonged QT
120J biphasic (Class II)
• history of torsades de pointes
May consider lower starting energy in AFI, very recent onset
AF, and those without suspected structural heart disease
Yes No


Arrange DCC when • Ibutilide (Class II)
Success?
sedation possible (Class II) • Procainamide (Class II)
• Amiodarone (Class II-III)
• Propafenone/flecainide Yes ➤ No


(Class II) (Only available
PO in the U.S. Slower in
onset.) • Reassess patient (Class • Increase energy level
indeterminate) (Class I-II) to max 360J
• Arrange further monophasic
workup, disposition
(Class II)


Success?

Yes No


• Reassess patient (Class Consider:
indeterminate) • Change pad place-
• Arrange further ment (Class II)
workup, disposition • Biphasic defibrillator
(Class II) (Class II)
• Medical
cardioversion**

* Only done in the ED after careful consideration of risks and benefits. Remember, 50% of patients with NOAF will convert
spontaneously within 24 hours.
** Success rate after failed electrical cardioversion not expected to be high

The evidence for recommendations is graded using the following scale. For complete definitions, see back page. Class I: Definitely recommended.
Definitive, excellent evidence provides support. Class II: Acceptable and useful. Good evidence provides support. Class III: May be acceptable,
possibly useful. Fair-to-good evidence provides support. Indeterminate: Continuing area of research.

This clinical pathway is intended to supplement, rather than substitute for, professional judgment and may be changed depending upon a
patient’s individual needs. Failure to comply with this pathway does not represent a breach of the standard of care.
Copyright ©2002 EB Practice, LLC. 1-800-249-5770. No part of this publication may be reproduced in any format
without written consent of EB Practice, LLC.

Emergency Medicine Practice 18 www.empractice.net • August 2002


Continued from page 14 The elderly and those with chest pain should usually be
• patients in whom there is no clinical suspicion of PE admitted. In some cases, patients may benefit from a brief
or MI; workup (including cardiac enzymes and echocardiography)
• patients in whom the AF converts in ED or the rate is for risk stratification in an observational-type unit.129 In this
controlled; and unit, patients might be rate controlled, started on anticoagu-
• patients for whom follow-up is ensured. lation (possibly LMWH), and discharged to see if spontane-

Cost-Effective Strategies For New-Onset Atrial Fibrillation/Atrial Flutter


1. Avoid “shotgun testing.” 4. Do not admit all NOAF patients to the CCU.
Not every patient who comes to the ED with NOAF requires a At some hospitals, all NOAF patients are admitted to the CCU
CBC, chemistries, toxicology screen, alcohol level, thyroid to “rule out MI.” This practice is not cost-effective, as the vast
function panel, d dimer, V/Q scan, coagulation studies, and a majority of patients with NOAF do not have acute coronary
bedside echo. Testing should be focused, based on patient syndrome. The overall incidence of MI with NOAF appears to
stability and findings of the history and physical examination. be 5%-15%. Most patients who do not otherwise appear at
An alcohol level and drug testing in the right clinical scenario risk for acute coronary syndrome (hemodynamically stable,
and a TSH in the elderly may be of some value. no ECG changes, no ongoing chest pain or significant
Caveat: Most of the findings suggesting that certain “low- symptoms, no prior MI or CHF, age < 65, no other cardiac risk
risk” patients with NOAF do not require cardiac enzymes or factors) can be admitted to a telemetry bed (and may not
evaluation for PE come from retrospective data; further require cardiac enzymes at all).
study is required. Do not assume that because a patient has Caveat: The evidence for this practice comes largely from
been drinking alcohol that the NOAF is only due to holiday retrospective data and must not take the place of good
heart syndrome. Unstable patients may benefit from having clinical judgment.
a bedside echo (after cardioversion) to rule out right heart
5. Avoid using the latest and greatest (and most expensive)
strain (suggestive of a PE) or acute valvular dysfunction.
drug on the market when what we have works as well
2. Do the simple stuff. or better.
When trying to diagnose a complex arrhythmia, consider Newer (read “expensive”) drugs for the treatment of AF are
simple, easily available tests. Use calipers to determine heavily promoted. However, just because an agent is newer or
whether the arrhythmia is regular or irregular. In the case of a more expensive does not necessarily mean it is better.
rapid ventricular response of unknown etiology, press the Magnesium, an inexpensive agent with few side effects, is as
speed button on the ECG to obtain a “double-speed” tracing effective as amiodarone for rate control and conversion of
that may demonstrate irregularity in the R-R interval. When AF—and you’ll never see it advertised in the latest journal.
atrial flutter is a possibility, move the chest leads to obtain a When new drugs are trialed, their use is often limited to a
Lewis lead or modified chest lead that could emphasize the very select group of patients. Only when used in the general
flutter waves. population do all of the complications come to light.
Caveat: Sometimes these interventions still leave you in
6. When considering elective conversion of stable AF in the
doubt. When the nature of the arrhythmia is uncertain,
ED, consider it again—and then if you still want to do it,
assume the worst and call for help.
consider electrical cardioversion over medical therapy.
3. Consider (very carefully) ED conversion and discharge Electrical cardioversion of NOAF in the stable patient
with close follow-up in some patients. has a reported success rate of 90%-100% vs. chemical
Not all patients with NOAF require admission, particularly therapy (which has a reported success of roughly 50%).
those with a very low suspicion of coronary artery disease, PE, Electrical cardioversion is effective and safe in the ED. While
or significant heart disease. If a patient spontaneously the use of electrical cardioversion is not without risks,
converts and does not appear to have another indication for antiarrhythmics are associated with significant
admission, it may be safe to send that patient home with proarrhythmic potential, and patients often require a
close follow-up. This is especially true in young patients with prolonged period of observation.
an obvious case of holiday heart syndrome. Patients who are Caveat: Some consider elective conversion of AF to be
already anticoagulated for another reason may be candidates outside the routine practice of emergency medicine. In
for early conversion in the ED with possible discharge. addition, procedural sedation (necessary for conversion of
Caveat: Most of the evidence for discharge of patients with the stable patient) carries a potential risk of aspiration and
AF is derived from retrospective data. Some authors airway compromise. Patients should be NPO for several
consider this practice risky. Also, given that up to 40%-70% hours prior to the procedure, something that’s not always
of patients with NOAF will spontaneously convert within practical in the ED. Before considering elective conversion
the first 24 hours, careful observation for conversion will in the ED, a conversation with the patient’s primary care
avoid the unnecessary use of electricity or antiarrhythmic physician and/or cardiologist is prudent. Discuss
drugs in a majority of patients. anticoagulation during this conversation. ▲

August 2002 • www.empractice.net 19 Emergency Medicine Practice


ous conversion occurs. Upon follow-up, an outpatient TEE hospitalization to facilitate a complete diagnostic evaluation.
could guide further therapy and intervention. In addition, conversion in the ED can greatly increase the
Early follow-up is mandatory for all NOAF patients length of stay in the ED, as the use of some of the chemical
being discharged. Patients must be given good discharge agents requires prolonged observation even after successful
instructions regarding the importance of follow-up and conversion (e.g., ibutilide). Because a large percentage of
signs or symptoms that should prompt a return to the ED. patients (40%-71%) with NOAF spontaneously convert in
the first 24 hours,70-73,105 some have questioned whether
Cutting-Edge/Controversy patients should be exposed to the risk of conversion agents
or electricity without a trial of observation.72 Finally, some
“Elective” Conversion In The ED ask whether patients with AF should be converted at
“Elective” conversion of certain stable patients with AF/AFl all,130,131 as the benefit of conversion and maintenance of
in the ED is gaining some momentum in the literature. It sinus rhythm is not always greater than rate control alone132
remains controversial, however. “Pro-ED converters” point (certainly a minority opinion). The Atrial Fibrillation
out that the literature supports early conversion to prevent Follow-up Investigation of Rhythm Management (AFFIRM)
atrial remodeling and increase the likelihood of maintaining study is a large, ongoing, prospective trial that addresses
normal sinus rhythm.12,13 They point out the (relative) safety this issue.133
of early conversion if the arrhythmia has been present for Both electrical and chemical interventions are reason-
less than 48 hours. Furthermore, early conversion can able options for elective cardioversion in the ED.23,24 With
reduce the need for anticoagulation and the associated risks either method of conversion, determine the possible need
of bleeding. Many of these patients may be discharged to for anticoagulation. Before proceeding, explain the risks and
outpatient testing and evaluation, which decreases costs. benefits and obtain informed consent.
Opponents argue that a stable patient should never be
converted electively in the ED. They emphasize the risk of Electricity
thromboembolism and argue that patients should be Despite all of the newer drugs for chemical cardioversion,
admitted for heparinization prior to cardioversion. They the traditional gold standard is DCC. DCC is relatively safe
also argue that the majority of patients with NOAF require and has a 90%-100% acute success rate in various settings,

Ten Pitfalls To Avoid


1. “The patient was intoxicated, so this was clearly holiday especially those causes associated with significant
heart syndrome. When his atrial fibrillation spontaneously morbidity and mortality.
converted, I sent him home with family to sober up.”
4. “I gave the patient the commonly quoted starting dose
Unfortunately, the patient you thought was “just drunk” was 68
of diltiazem, a 20 mg bolus. It is unfortunate that she got
years old, a diabetic, and a longtime smoker who told the
profoundly hypotensive, seized, and arrested, but I did
nurse (but not you) that he had “crushing chest pain.” If you
nothing wrong.”
had pulled his old ECG, you would have noted some
Had you considered that the patient to whom you gave the
disturbing new changes. Although he converted shortly after
diltiazem weighed no more than 80 pounds soaking wet, you
arrival, the signs and symptoms of ischemia associated with
might have started with a lower dose and titrated it slowly. You
significant cardiac risk factors mandated admission.
can always give more drug, but it’s hard to take it back. Also,
2. “The ibutilide worked great. The patient felt much better you might have given calcium pretreatment or at least had it
and wanted to go home immediately.” at the bedside prior to pushing the diltiazem when the
While ibutilide works to convert AF/AFl approximately patient’s starting blood pressure was 100/60 mmHg. Always
40%-50% of the time, it has significant risks—most notably have the nurses give these medications slowly over a few
torsades de pointes and other ventricular tachyarrhythmias. minutes (and never “push” them) to minimize hypotension.
The pharmacokinetics of the drug require a four-hour 5. “When the diltiazem didn’t give a good response, I
period of monitoring after use. decided to try another drug. I believe her complete heart
3.“This was a case of routine atrial fibrillation in a young block was from the acute coronary syndrome she was
female. Yes, she felt slightly short of breath, but her heart rate having, not what I did.”
was 160. I controlled her rate and even admitted her to the A classic blunder—giving an IV ß-blocker soon before or after
hospital. Her physician said he would start anticoagulation an IV calcium-channel blocker. It is safe to give one of these
therapy in the morning if she did not convert.” two classes of drugs IV—cautiously—if the patient is on an
Had you only asked about the long plane ride she had just oral version of the other class, but not both intravenously.
been on, and the fact that she was recently diagnosed with 6. “I thought the patient might be having acute coronary
ovarian cancer, you might have more strongly considered syndrome and therefore used a ß-blocker for its beneficial
the diagnosis of pulmonary embolism. Deal with the atrial effects. The patient just seemed to decompensate around
fibrillation, but also consider the differential diagnosis, Continued on page 21

Emergency Medicine Practice 20 www.empractice.net • August 2002


including the ED.23,24 (The sustained conversion rates, the ED. Eighty of the patients, most of whom had failed
particularly in certain groups of patients, are much lower.) chemical conversion in the same visit, had electrical
The downsides of the procedure include the need for cardioversion, with an 89% success rate. Emergency
sedation and the attendant possibility of respiratory failure. physicians performed 93% of these electrical cardioversions.
The emergency physician must also be prepared to intubate There were no reported complications, and most of the
and manage induced arrhythmias, although most of these patients were discharged. The authors reported this practice
are transient and resolve spontaneously. to be “safe and effective” but admitted that this needs to be
To perform DCC, take the usual precautions with validated in a prospective trial. Also, it should be noted that
sedation. Place the patient on a monitor, oxygen, pulse the follow-up in this study was not ideal. An abstract also
oximeter, and obtain good IV access. Have airway and reports on the electrical cardioversion of patients in the ED.24
cardiac resuscitation equipment readily available. Adminis- This study, which appears to be retrospective, reported 315
ter adequate sedation whenever possible, as it is considered out of 350 successful electrical cardioversions by emergency
a travesty to cardiovert someone who is completely awake physicians, with an overall 2.5% rate of complications, most
(the exception being the moribund patient, in whom of which were reported as minor. As of this printing, a
sedation may be a luxury neither the physician nor the MEDLINE search has not revealed this study published in
patient can afford). Once a state of relaxation is obtained, its entirety, which limits further interpretation of the results.
cardiovert with paddles in the anterior-posterior position
starting with 200J for AF and 100J for AFl using a standard Drugs
monophasic defibrillator. If using a biphasic defibrillator, Several classes of antiarrhythmic drugs have potential for
begin with 120J for AF and 70J for AFl. If maximal energy use in the ED for conversion of AF. (See Table 9 on page 22.)
levels do not convert the patient, change the paddle As of this time, ibutilide is the only IV medication with FDA
positions to the anterior-lateral position and reattempt. approval for acute termination of AF and AFl. However,
One study and one abstract have been published that many other drugs have been studied for this indication,
address electrical cardioversion of patients in the ED. The including a few agents that are not available intravenously
study, published in Annals of Emergency Medicine by Michael in the United States (dofetilide, sotalol, flecainide, and
et al,23 was a retrospective study of 289 conversions of AF in propafenone). Comparison between the different antiar-

Ten Pitfalls To Avoid (continued)

that same time—an unfortunate coincidence.” patient and sent her home. She was just one of the
While there are some advantages to the use of a ß-blocker unfortunate people who had a thromboembolic event.”
in NOAF in the setting of acute coronary syndrome or Although there is some suggestion in the literature that the
thyrotoxicosis, remember the drug’s contraindications. Had method you used might be reasonable, the data suggest that
you noticed the significant wheezing on examination and anticoagulation would be required even with a negative TEE in
stopped to think that the patient was frequently on home this situation. If someone has been in AF for more than 48
nebulizers for asthma, you might have given a calcium- hours, TEE may not show a clot, but there may still be as high
channel blocker instead (or at least given a short-acting as a 2% incidence of thromboembolism after conversion.
ß-blocker such as esmolol). Patients with AF for greater than 48 hours (and sometimes
even less) likely benefit from anticoagulation therapy at least
7. “It was clearly irregular. I’m positive it was atrial
after the procedure, and possibly before.
fibrillation, and that’s why I gave diltiazem. I was quite
surprised when the heart rate actually got worse.” 9. “She didn’t look bug-eyed to me.”
Had you looked more carefully at the ECG, you might have In the elderly, thyrotoxicosis can present very atypically,
noticed the delta wave and prolonged QRS duration that without the common findings that usually occur in younger
suggested a reentrant pathway. Consider an accessory patients. A TSH is a reasonable screening test in the elderly
pathway in AF with any of the following: a wide complex with NOAF.
irregular rhythm (especially if the complexes vary within the
10. “I gave diltiazem because the patient was not
ECG), a very fast rhythm (RR > 250), a history of an accessory
anticoagulated, and I did not want to cardiovert her
pathway (don’t miss this one), and a younger patient with
and cause a stroke.”
NOAF. Giving AV nodal blocking agents is contraindicated
The patient was diaphoretic, with decreased mental status,
when an accessory pathway is present. Theses drugs block the
and had a blood pressure of 60/palp and an irregular
AV node, thus making the conduction down the accessory
heart rate of 160. That defines unstable. Electrical
pathway worse, with a resultant increase in ventricular rate. A
cardioversion is the initial treatment of choice. While it
safer choice might be procainamide, amiodarone, or
does carry a risk of thromboembolism, the risk of death
cardioversion, depending on the situation.
from sustained shock is greater. After the unstable patient
8. “Sure her AF was one week old, but I did a TEE that cardioverted, start heparin as soon as possible if there is
showed no left atrial clot, and therefore I cardioverted the no significant contraindication. ▲

August 2002 • www.empractice.net 21 Emergency Medicine Practice


rhythmic agents is difficult, because the published studies conversion in the typical ED setting at this time appear to be
vary greatly in quality and applicability to patients in the ibutilide, amiodarone, procainamide, and possibly the Class
ED. Overall success rates average around 50% depending IC agents (flecainide and propafenone), although the latter
on the drug used, patient characteristics, study design, and are only available PO in the United States. Although the oral
mode of delivery. Most trials are small, and thus conclusions forms have been used and studied for the conversion of AF,
regarding safety and efficacy are questionable. Placebo arms the ED use of these agents is not widespread. Some argue
are not always available in these trials, thus limiting easy that the initiation of the oral forms of these drugs warrants
comparison of agents. Remember that at least 40%-71% of prolonged observation for proarrhythmic effects. Elective
patients with AF convert spontaneously within the first 24 chemical conversion by the ED physician is best performed
hours,70-73,105 whereas patients with chronic AF are unlikely in consultation with the patient’s primary care physician or
to convert spontaneously. Therefore, the duration of AF in cardiologist, as these drugs can have serious side effects.
study patients (which can vary greatly from trial to trial) Ibutilide is worth mentioning, as its use in the ED
would significantly affect the overall reported success of the setting has been suggested due to its short half-life. The
conversion agent. Furthermore, some studies only include major risk of the drug is the ability to induce torsades de
patients who have already failed a previous attempt at pointes in up to 8% of patients, which can occur up to
conversion, and thus the reported success may appear several hours after the drug is given.39,134-136 Thus, careful
very low. monitoring for four hours after administration—even with
The realistic chemical options (if there are any) for successful conversion—must be mandatory. Amiodarone has

Table 9. Elective Conversion Agents.

Dosing Contraindications/Cautions Comments


Electrical Start with 200J monophasic, 120J Sedation required Consider lower starting doses in
cardioversion biphasic synchronized (if unstable, NOAF, AFl, and in patients unlikely
maximal initial doses are likely indicated) to have structural heart disease
Class IA
• Procainamide 100 mg IV q 5-10 minutes to maximum Stop drug if conversion, prolonged Rate control agent should precede
of 1000 mg or as a 20 mg/kg IV infusion QT/QRS, torsades de pointes, its use given a risk of increased
to a maximum of 20 mg/kg hypotension conduction through the AV node
• Quinidine 200-300 mg sulfate PO, followed in 1-2 Not used much currently
hours by 400 mg
Class IC High proarrhythmic effects and
negative inotropic effects in those
with structural heart disease
• Flecainide 2 mg/kg IV over 10 minutes* Highest reported success rate
or 300 mg PO times 1 among various drugs
• Propafenone 2 mg/kg IV over 10 minutes*
or 600 mg PO times 1
Class III
• Ibutilide 0.01 mg/kg IV over 10 minutes (max, Hypokalemia, hypomagnesemia, Conversion rate better for AFl than
1 mg). Repeat times 1 if no response LV dysfunction, prolonged QT. AF; expensive ($200 per 1 mg); very
10 minutes after first dose completed Induction of torsades de pointes short half life, but use warrants
as high as 8% observation for at least four hours
for torsades de pointes
• Dofetilide 0.5 mg PO bid. Not available IV Similar to ibutilide ED use unlikely as FDA has
restricted access in United States.
Use of drug likely warrants 72-hour
period of observation
• Amiodarone 150 mg IV over 10 minutes followed by Hypotension may occur with IV use May be used for rate control,
infusion of 1 g over 6 hours. May repeat conversion, and maintenance;
bolus if needed. Or, 30 mg/kg PO times 1 low proarrhythmic effects
• Sotalol 1.5 mg/kg IV over 30 minutes* Significant reactive airway disease Effective for rate control and
conversion; however, generally
poor conversion efficacy. Blocks
accessory pathways
*Not currently available in the United States

Emergency Medicine Practice 22 www.empractice.net • August 2002


the advantage of having rate control effects as well as some tration of the drug in WPW.115 In Europe, propafenone and
success at the conversion of AF/AFl. Of note, however, are sotalol are used, but these drugs are not available in IV form
its poor success rate compared to other drugs and its slow in the United States.138
onset, which limit its utility. Despite the lack of FDA
approval for acute conversion in AF, procainamide has been Summary
used extensively and can be given easily IV, which makes it
a reasonable option in the ED. Because it can increase Atrial fibrillation and its close counterpart, atrial flutter, are
conduction through the AV node, an AV nodal blocking agent commonly encountered in the ED, and their incidence will
should be used prior to its use as it could induce an increased only continue to rise. It is imperative for emergency
ventricular response if given alone. physicians to keep abreast of the latest literature regarding
So which drug is best? That’s a hard question to the management of these patients. Cardiovert the truly
answer, as the reported success of each drug often varies unstable patient, but anticipate possible failure or short-
largely from study to study, making it difficult to place the lived effects. For stable patients, be familiar with the
drug’s usefulness in context. The bottom line is that there is appropriate use of rate-control medications. Ventricular rate
not likely a single drug of choice at this time for the acute control remains the most commonly required ED interven-
conversion of AF. tion in patients with NOAF. Anywhere from 40%-70% of
patients with NOAF will spontaneously convert without
Special Circumstances electricity or toxic drugs.
The role of anticoagulation regarding patients with
Accessory Pathways NOAF of less than 48 hours is hotly debated. Although
The management of pre-excitation syndromes like Wolff- there are no large, prospective trials, some data suggest that
Parkinson-White syndrome is particularly challenging patients can be risk stratified to determine the need for
in patients with AF.137 The short refractory period of anticoagulation and admission. Some literature suggests
these accessory pathways can result in extremely fast that stable patients with AF of short duration may be safely
ventricular rates. converted in the ED; however, this practice can hardly be
So when should one consider WPW in a patient with considered routine. Admission and discharge decisions are
AF? Certainly a history of WPW will give it away, but there best made on an individual basis and should incorporate
are other findings that should raise suspicion. (See Figure 3 factors such as age, likelihood of a serious underlying
and Figure 4 on page 6.) AF with bizarre wide complexes of etiology, comorbidities, clinical stability, and access to early
different morphologies, a very rapid ventricular rate (RR follow-up. Although many questions regarding the manage-
interval > 250), and younger patients should all be consid- ment of AF in the ED remain unanswered, “the beat goes
ered for an underlying accessory pathway. Look for the on”—even if it is irregular. ▲
presence of a delta wave, which is a slurred, slow-rising
onset of the QRS complex.138 References
Do not use AV nodal blocking agents in anyone you suspect
as having an accessory pathway (unless you pretreat with Evidence-based medicine requires a critical appraisal of the
procainamide).31,138 These drugs can actually increase the literature based upon study methodology and number of
ventricular response in patients with WPW and result in subjects. Not all references are equally robust. The findings
ventricular fibrillation.31 In patients with accessory path- of a large, prospective, randomized, and blinded trial
ways and rapid AF, ß-blockers, CCBs, and digoxin are all should carry more weight than a case report.
contraindicated as sole treatment agents. To help the reader judge the strength of each reference,
Synchronized electrical cardioversion is the treatment pertinent information about the study, such as the type of
of choice in unstable patients with accessory pathways, study and the number of patients in the study, will be
and some authors recommend it as first-line treatment included in bold type following the reference, where
even in stable patients. For reasons discussed earlier in available. In addition, the most informative references cited
this article, starting at 200J-360J synchronized monophasic in the paper, as determined by the authors, will be noted by
shock is appropriate. an asterisk (*) next to the number of the reference.
For stable patients, procainamide followed by an AV 1.* ACC/AHA/ESC guidelines for the management of patients with
atrial fibrillation: executive summary a report of the American College
nodal blocking agent is also reasonable. Procainamide slows
of Cardiology/American Heart Association Task Force on practice
conduction through accessory pathways but can increase guidelines and the European Society of Cardiology Committee for
conduction through the AV node and thus should be practice guidelines and policy conferences. Circulation
2001;104(17):2118-2150. (Evidence-based review)
followed by an AV nodal blocking agent. Procainamide can
2. Adult advanced cardiac life support. ACLS Provider Manual. Dallas:
be loaded in 50-100 mg boluses IV every 5-10 minutes or by American Heart Association; 2001. (Textbook)
an infusion at up to 30 mg/min. No more than 17 mg/kg 3. Go AS, Hylek EM, Phillips KA, et al. Prevalence of diagnosed atrial
fibrillation in adults: national implications for rhythm management
total should be given, and it should be stopped if the QRS
and stroke prevention: the anticoagulation and risk factors in atrial
widens greater than 50%, hypotension occurs, or the patient fibrillation (ATRIA) study. JAMA 2001;285(18):2370-2375. (Cross-
converts to sinus rhythm. Amiodarone might have a role in sectional cohort; 17,974 patients)
4. Kannel WB, Abbott RD, Savage DD, et al. Epidemiologic features of
the treatment of accessory pathways, but a few case reports
atrial fibrillation: the Framingham study. N Engl J Med 1982;306:1018-
have reported ventricular rate acceleration with IV adminis- 1022. (Observational cohort; 2325 men, 2866 women)

August 2002 • www.empractice.net 23 Emergency Medicine Practice


5. Lin HJ, Wolf PA, Kelly-Hayes M, et al. Stroke severity in atrial 29. Gillis AM, Klein GJ, MacDonald RG. Investigation of the patient with
fibrillation: the Framingham study. Stroke 1996;27(10):1760-1764. atrial fibrillation. Can J Cardiol 1996;12(supplA):12A-13A. (Consensus
(Observational cohort; 501 strokes in 5070 patients) conference)
6. Seidl K, Hauer B, Schwick NG, et al. Risk of thromboembolic events in 30. Accardi AJ, Miller R, Holmes JF. Enhanced diagnosis of narrow
patients with atrial flutter. Am J Cardiol 1998;82(5):580-583. (Follow-up complex tachycardias with increased electrocardiograph speed. J
study; 191 patients) Emerg Med 2002 Feb;22(2):123-126. (Prospective, comparative; 45
7. Al-Khatib SM, Pieper KS, Lee KL, et al. Atrial fibrillation and mortality ECGs)
among patients with acute coronary syndromes without ST-segment 31. McGovern B, Garan H, Ruskin JN. Precipitation of cardiac arrest by
elevation: Results from the PURSUIT Trial. Am J Cardiol 2001;88(1):76- verapamil in patients with Wolff-Parkinson-White Syndrome. Ann
79. (Multicenter, randomized, controlled trial) Intern Med 1986;104:791-794. (Case series; 5 patients)
8. Lok NS, Lau CP. Presentation and management of patients admitted 32. Goldman MJ. Principles of Clinical Electrocardiography. Los Altos, CA:
with atrial fibrillation: a review of 291 cases in a regional hospital. Lange Medical Publications; 1982. (Textbook)
Internat J Cardiol 1995;48:271-278 (Retrospective; 291 patients) 33. Herbert ME, Votey SR. Adenosine in wide-complex tachycardia. Ann
9. Varriale P, Ramaprasad S. Aminophylline induced atrial fibrillation. Emerg Med 1997;29(1):172-174. (Review)
Pac Clin Electrophysiol 1993;16(10):1953-1955. (Case report; 3 patients) 34. Krahn AD, Klein GJ, Kerr CR, et al. How useful is thyroid function
10. Okada M. The cardiac rhythm in accidental hypothermia. J testing in patients with recent-onset atrial fibrillation? Arch Intern Med
Electrocardiol 1984;17(2):123-128. (Retrospective; 60 ECGs) 1996;156(19):2221-2224. (726 patients)
11. Lowenstein SR, Gabow PA, Cramer J, et al. The role of alcohol in new- 35. Trivalle C, Doucet J, Chassagne P , et al. Differences in the signs and
onset atrial fibrillation. Arch Intern Med 1983;143:1882-1885. (Retro- symptoms of hyperthyroidism in older and younger patients. J Am
spective chart review; 40 patients) Geriatr Soc 1996;44(1):50-53. (Prospective cohort; 84 patients)
12. Wijffels M, Kirchhof C, Dorland R, et al. Atrial fibrillation begets atrial 36. Sawin CT, Geller A, Wolf PA, et al. Low serum thyrotropin concentra-
fibrillation: a study in awake chronically instrumented goats. tions as a risk factor for atrial fibrillation in older persons. N Engl J Med
Circulation 1995;92:1954-1968. (Animal study) 1994;331(19):1249-1252. (Observational cohort; 2007 patients)
13. Goette A, Honeycutt C, Langberg JJ. Arrhythmias/Pacing: electrical 37. Osman F, Gammage MD, Sheppard MC, et al. Cardiac dysrhythmias
remodeling in atrial fibrillation: time course and mechanisms. and thyroid dysfunction: the hidden meance? J Clin Endocrin Metabol
Circulation 1996;94(1):2968-2974. (Animal study, basic science report) 2002;87(3):963-967. (Retrospective cohort; 7209 patients)
14. Berger M, Schweitzer P. Timing of thromboembolic events after 38. Ellenbogen KA, Clemo HF, Stambler BS, et al. Efficacy of ibutilide for
electrical cardioversion of atrial fibrillation or flutter: a retrospective termination of atrial fibrillation and flutter. Am J Cardiol 1996;78(suppl
analysis. Am J Cardiol 1998;82(12):1545-1547. (Retrospective review; 92 8A):42-45. (Review)
patients with embolism) 39. Granberry MC. Ibutilide: a new class III antiarrhythmic agent. Am J
15. Frykman V, Frick M, Jensen-Urstad M, et al. Asymptomatic versus Health Syst Pharm 1998;55(3):255-260. (Formulary review)
symptomatic persistent atrial fibrillation: clinical and noninvasive 40. French JH, Thomas RG, Siskind AP, et al. Magnesium therapy in
characteristics. J Intern Med 2001;250(5):390-397. (Prospective; 282 massive digoxin intoxication. Ann Emerg Med 1984 Jul;13(7):562-566.
patients) (Case report)
16. Lin HJ, Wolf PA, Benjamin EJ, et al. Newly diagnosed atrial fibrillation 41. Goldberg RJ, Yarzebski J, Lessard D, et al. Recent trends in the
and acute stroke: The Framingham study. Stroke 1995;26:1527-1530. incidence rates of death rates from atrial fibrillation complicating
(Observational cohort; 115 strokes associated with atrial fibrillation) initial acute myocardial infarction: a community-wide perspective. Am
17. Biblo LA, Yuan Z, Quan KJ, et al. Risk of stroke in patients with atrial Heart J 2002;143(3):519-527. (Longitudinal study, 2596 patients)
flutter. Am J Cardiol 2001;87(3):346-349. (Cohort analysis; 749,988 42. Friedman HZ, Goldberg SF, Bonema JD, et al. Acute complications
patients) associated with new-onset atrial fibrillation. Am J Cardiol 1991;67:437-
18. Bikkina M, Alpert MA, Mulekar M, et al. Prevention of intraatrial 439. (Prospective; 98 patients)
thrombus in patients with atrial flutter. Am J Cardiol 1995;76(3);186-189 43.* Mulcahy B, Coates WC, Henneman PL, et al. New-onset atrial
(24 patients with atrial flutter) fibrillation: when is admission medically justified? Acad Emerg Med
19. Abarbanell NR, Marcotte, MA, Schaible BA, et al. Prehospital 1996;3(2):114-119. (Retrospective; 216 patients)
management of rapid atrial fibrillation: recommendations for 44. Friedman HZ, Weber-Bornstein N, Deboe SF, et al. Cardiac care unit
treatment protocols. Am J Emerg Med 2001;19(1):6-9. (Retrospective trip admission criteria for suspected acute myocardial infarction in new-
sheet review; 33 patients with AF) onset atrial fibrillation. Am J Cardiol 1987;59:866-869. (Retrospective; 45
20. Wang HE, O’Conner RE, Merargel, RE, et al. The use of diltiazem for patients)
treating rapid atrial fibrillation in the out-of-hospital setting. Ann 45. Shlofmitz RA, Hirsch BE, Meyer BR. New-onset atrial fibrillation: is
Emerg Med 2001;37(1):38-45. (Retrospective; 43 patients who received there a need for emergent hospitalization? J Gen Intern Med 1986;1:139-
diltiazem, 27 historical controls) 142. (Retrospective; 97 patients)
21. Bertini G, Conti A, Fradella G, et al. Propafenone versus amiodarone in 46.* Zimetbaum PJ, Josephson ME, McDonald MJ, et al. Incidence and
field treatment of primary atrial tachydysrhythmias. J Emerg Med predictors of myocardial infarction among patients with atrial
1990;8:15-20. (Prospective; 39 patients, 28 with AF) fibrillation. J Am Coll Cardiol 2000;36:1223-1227. (Prospective cohort;
22. Elam K, Bolar-Softich KL. Dilemmas in the acute pharmacologic 255 patients)
treatment of uncontrolled atrial fibrillation. Am J Emerg Med 47. Kline JA, Johns KL, Colucciello SA, et al. New diagnostic tests for
1997;15:418-419. (Uncontrolled, nonblinded; 26 patients) pulmonary embolism. Ann Emerg Med 2000 Feb;35(2):168-180.
23.* Michael JA, Stiell IG, Agarwal S, et al. Cardioversion of paroxysmal (Prospective, multicenter study, review)
atrial fibrillation in the emergency department. Ann Emerg Med 48.* Silverman DI, Manning WJ. Role of echocardiography in patients
1999;33(4):379-387. (Retrospective consecutive cohort; 289 patients) undergoing elective cardioversion of atrial fibrillation. Circulation
24. Darawshe A, Malatskey-Seligmann L, Freedberg NA, et al. Elective 1998;98(5):479-486. (Review)
and urgent electrical cardioversion of atrial fibrillation in the 49. Moreyra E, Finkelhor RS, Cebul RD. Limitations of transesophageal
emergency department: its safety and efficacy. Ann Emerg Med echocardiography in the risk assessment of patients before
2000;36(4):S73 (Abstract; 350 episodes) nonanticoagulated cardioversion from atrial fibrillation and flutter: an
25. Clair WK, Wilkinson WE, McCarthy EA, et al. Spontaneous occurrence analysis of pooled trials. Am Heart J 1995;129:71-75. (Systematic
of symptomatic paroxysmal atrial fibrillation and paroxysmal literature review; 374 patients)
supraventricular tachycardia in untreated patients. Circulation 50. Manning WJ, Silverman DI, Gordon SPF, et al. Cardioversion from
1993;87(4):1114-1122. (Consecutive series; 150 patients) atrial fibrillation without prolonged anticoagulation with the use of
26. Page RL, Wilkinson WE, Clair WK, et al. Arrhythmia/EP/Pacing: transesophageal echocardiography to exclude the presence of atrial
Asymptomatic arrhythmias in patients with symptomatic paroxysmal thrombi. N Engl J Med 1993;328:750-755. (Prospective; 94 patients)
atrial fibrillation and paroxysmal supraventricular tachycardia. 51. Murray RD, Deitcher SR, Shah A, et al. Potential clinical efficacy and
Circulation 1994;89(1):224-227. (Observational; 8 patients with PAF) cost benefit of transesophageal echocardiography-guided low-
27. Sykes D, Dewar R, Mohanaruban K, et al. Measuring blood pressure in molecular-weight heparin (enoxaparin) approach to antithrombotic
the elderly: does atrial fibrillation increase observer variability? BMJ therapy in patients undergoing immediate cardioversion from atrial
1990;300:162-163. (Prospective variability study; 100 patients) fibrillation. J Am Soc Echocardiogr 2001;14(3):200-208. (Clinical and cost
28. Varney SM, Manthey DE, Culpepper VE, et al. A comparison of oral, analysis)
tympanic, and rectal temperature measurement in the elderly. J Emerg 52. Black IW, Hopkins AP, Lee LCL, et al. Evaluation of transesophageal
Med 2002 Feb;22(2):153-157. (Comparative; 95 patients) echocardiography before cardioversion of atrial fibrillation and flutter

Emergency Medicine Practice 24 www.empractice.net • August 2002


in nonanticoagulated patients. Am Heart J 1993;126:375-381. (Prospec- Med 1997;30(3):354-355. (Editorial)
tive; 40 patients) 77. Schwinger RHG, Bohm M, Erdmann E. Negative inotropic properties
53. Corrado G, Tadeo G, Beretta S, et al. Atrial thrombi resolution after of isradipine, nifedipine, diltiazem, and verapamil in diseased human
prolonged anticoagulation in patients with atrial fibrillation: a myocardial tissue. J Cariovasc Pharmacol 1990;15:892-899. (In vitro
transesophageal echocardiographic study. Chest 1999;115(1):140-143. study)
(Prospective; 11 patients with initial clot) 78.* Phillips BG, Gandhi AJ, Sanoski CA, et al. Comparison of intravenous
54.* Klein AL, Grimm RA, Murray RD, et al. Use of transesophageal diltiazem and verapamil for the acute treatment of atrial fibrillation
echocardiography to guide cardioversion in patients with atrial and atrial flutter. Pharmacotherapy 1997;17(6):1238-1245. (Prospective,
fibrillation. N Engl J Med 2001;344(19):1411-1420. (Prospective; 1222 randomized, double-blind, crossover; 17 patients)
patients) 79. Cheiman DM, Shea BF, Kelly RA. Treatment of supraventricular
55. Jaber WA, Prior DL, Thamilarasan M, et al. Efficacy of anticoagulation tachyarrhythmias with intravenous calcium channel blockers: are
in resolving left atrial and left atrial appendage thrombi: a subtle differences worth the cost? Pharmacotherapy 1996;16(5):861-868.
transesophageal echocardiographic study. Am Heart J 2000;140(1):150- (Review)
156. (Retrospective; 174 patients with thrombi) 80. Ellenbogen KA, Dias VC, Cardello FP, et al. Safety and efficacy of
56. Botto GL, Politi A, Bonini W, et al. External cardioversion of atrial intravenous diltiazem in atrial fibrillation or atrial flutter. Am J Cardiol
fibrillation: role of paddle position on technical efficacy and energy 1995;75:45-49. (Open-label, dose titration study; 84 patients)
requirements. Heart 1999;82(6):726-730. (Prospective; 301 patients) 81.* Haft JI, Habbab MA. Treatment of atrial arrhythmias—effectiveness of
57. Elhendy A, Gentile F, Khandheria BK, et al. Predictors of unsuccessful verapamil when preceded by calcium infusion. Arch Intern Med
electrical cardioversion in atrial fibrillation. Am J Cardiol 2002;89(1):83- 1986;146:1085-1089. (Sequential, nonrandomized, nonblinded; 59
86. (Prospective; 692 patients) pretreated patients)
58. Kerber RE, Jensen SR, Grayzel J, et al. Elective cardioversion: influence 82. Dolan DL. Intravenous calcium before verapamil to prevent
of paddle-electrode location and size on success rates and energy hypotension. Ann Emerg Med 1991;20(5):588-589. (Editorial)
requirements. N Engl J Med 1981;305(12):658-662. (Prospective; 173 83. Kuhn M, Schriger DL. Low-dose calcium pretreatment to prevent
patients) verapamil-induced hypotension. Am Heart J 1992;124(1):231-232.
59. Ewy GA. Optimal technique for electrical cardioversion of atrial (Retrospective, nonrandomized; 18 treated patients, 117 controls)
fibrillation. Circulation 1992;86(5):1645-1647. (Editorial comment) 84.* Goldenberg IF, Lewis WR, Dias VC, et al. Intravenous diltiazem for the
60. Ricard P, Levy S, Trigano J, et al. Prospective assessment of the treatment of patients with atrial fibrillation or flutter and moderate to
minimum energy needed for external electrical cardioversion of atrial severe congestive heart failure. Am J Cardiol 1994;74:884-889.
fibrillation. Am J Cardiol 1997;79(6):815-816. (Prospective; 198 patients) (Prospective, randomized, placebo-controlled; 37 patients)
61. Joglar JA, Hamdan MH, Ramaswamy K, et al. Initial energy for 85. Vinson DR, Burke TF, Sung HM. Rapid reversal of life-threatening
elective external cardioversion of persistent atrial fibrillation. Am J diltiazem-induced tetany with calcium chloride. Ann Emerg Med
Cardiol 2000;86(3):348-350. (Prospective; 64 patients) 1999;34(5):676-678. (Case report)
62. Greaves K, Crake T. Cardiac troponin T does not increase after 86. Hwang MH, Danoviz J, Pacold I, et al. Double-blind crossover
electrical cardioversion for atrial fibrillation or atrial flutter. Heart randomized trial of intravenously administered verapamil: its use for
1998;80(3):226-228. (Prospective; 15 patients) atrial fibrillation and flutter following open heart surgery. Arch Intern
63. Saliba W, Juratli N, Chunk MK, et al. Higher energy synchronized Med 1984;144:491-494. (Prospective, double-blind crossover; 14
external direct current cardioversion for refractory atrial fibrillation. J patients)
Am Coll Cardiol 1999;34:2031-2034. (Retrospective, comparative; 55 87. Salerno DM, Dias VC, Kleiger RE, et al. Efficacy and safety of
patients) intravenous diltiazem for treatment of atrial fibrillation and atrial
64. Bjerregaard P, El-Shafei A, Janosik DL, et al. Double external direct- flutter. Am J Cardiol 1989;63:1046-1051. (Prospective, double-blind,
current shocks for refractory atrial fibrillation. Am J Cardiol parallel, randomized, placebo-controlled; 113 patients)
1999;83(6):972-974. (Prospective; 24 patients) 88. Ellenbogen KA, Dias VC, Plumb VJ, et al. A placebo-controlled trial of
65. Oral H, Souza JJ, Michaud GF, et al. Facilitating transthoracic continuous intravenous diltiazem infusion for 24-hour heart rate
cardioversion of atrial fibrillation with ibutilide pretreatment. N Engl J control during atrial fibrillation and atrial flutter: a multicenter study. J
Med 1999;340(24):1849-1854. (Prospective, randomized; 100 patients) Am Coll Cardiol 1991;18(4):891-897. (Randomized, double-blinded,
66. Mittal S, Ayati s, Stein KM, et al. Transthoracic cardioversion of atrial placebo controlled; 47 patients)
fibrillation: comparison of rectilinear biphasic versus damped sine 89. Segal JB, McNamara RL, Miller MR, et al. The evidence regarding the
wave monophasic shocks. Circulation 2000;101:1282-1287. (Prospective; drugs used for ventricular rate control. J Fam Pract 2000;49(1):47-59.
165 patients) (Evidence-based review; 45 articles)
67. Niebauer MJ, Chung MK, Wilkoff BL, et al. Success rate of the 90.* Schreck DM, Rivera AR, Tricarico VJ. Emergency management of atrial
rectilinear biphasic waveform in atrial cardioversion in a large cohort fibrillation and flutter: intravenous diltiazem versus intravenous
of patients. Oral Presentation at the 73rd Scientific Session of the digoxin. Ann Emerg Med 1997;29(1):135-140. (Prospective, randomized,
American Heart Association, New Orleans, LA (November 14,2000). open label; 30 patients)
Circulation 2000;102(suppl):II-574 (Abstract; 530 patients) 91. Innes GD, Vertesi L, Dillon EC, et al. Effectiveness of verapamil-
68. Chalasani P, Cambre S, Silverman ME. Direct-current cardioversion for quinidine versus digoxin-quinidine in the emergency department
the conversion of atrial flutter. Am J Cardiol 1996;77(8):658-660. treatment of paroxysmal atrial fibrillation. Ann Emerg Med
(Retrospective; 98 episodes in 85 patients) 1997;29(1):126-134. (Prospective, double-blind, randomized; 44
69. Pinski SL, Sgarbossa EB, Ching E, et al. A comparison of 50-J versus patients)
100-J shocks for direct-current cardioversion of atrial flutter. Am Heart J 92. Tisdale JE, Padhi DI, Goldberg DA, et al. A randomized, double-blind
1999;137(3):439-442. (Prospective, nonrandomized; 330 patients) comparison of intravenous diltiazem and digoxin for atrial fibrillation
70. Falk RH, Knowlton AA, Bernard SA, et al. Digoxin for converting after coronary artery bypass surgery. Am Heart J 1998;135(5):739-747.
recent-onset atrial fibrillation to sinus rhythm. Ann Intern Med (Prospective; 40 patients)
1987;106:503-506. (Prospective; 36 patients) 93. Wattanasuwan N, Khan, IA, Mehta NJ, et al. Acute ventricular rate
71. Jordaens L, Trouerbach J, Calle R, et al. Conversion of atrial fibrillation control in atrial fibrillation: IV combination of diltiazem and digoxin
to sinus rhythm and rate control by digoxin in comparison to placebo. vs. IV diltiazem alone. Chest 2001;119(2):502-506. (Prospective,
Eur Heart J 1997;18:643-648. (Prospective; 40 patients) randomized, open label; 52 patients)
72. Donovan KD, Power BM, Hockings BEF, et al. Intravenous flecainide 94. Delle Karth G, Geppert A, Neunteufl T, et al. Amiodarone versus
versus amiodarone for recent-onset atrial fibrillation. Am J Cardiol diltiazem for rate control in critically ill patients with atrial
1995;75:693-697. (Prospective, randomized, double-blind; 98 patients) tachyarrhythmias. Crit Care Med 2001;29(6):1149-1153. (Prospective; 60
73.* Ergene U, Ergene O, Fowler J, et al. Must antidysrhythmic agents be patients)
given to all patients with new-onset atrial fibrillation? Am J Emerg Med 95. Barbarash RA, Bauman JL, Lukazewski AA, et al. Verapamil infusion
1999;17:659-662. (Observational cohort; 66 patients) in the treatment of atrial tachyarrhythmias. Crit Care Med
74. No authors listed. Drugs for cardiac arrhythmias. Med Lett 1986;14(10):886-888. (Retrospective; 10 patients)
1996;38(982):75-82. (Systematic review) 96. Blackshear JL, Stambler BS, Strauss WE, et al. Control of heart rate
75. Bohm M, Schwinger RHG, Erdmann E. Different cardiodepressant during transition from intravenous to oral diltiazem in atrial
potency of various calcium antagonists in human myocardium. Am J fibrillation or flutter. Am J Cardiol 1996;78:1246-1250. (Open label,
Cardiol 1990;65:1039-1041. (In vitro study) therapeutic trial; 40 patients)
76. Zemenick RB. Verapamil or diltiazem for acute rate control. Ann Emerg 97. Kuhn M, Schriger DL. Verapamil administration to patients with

August 2002 • www.empractice.net 25 Emergency Medicine Practice


contraindication: is it associated with adverse outcomes? Ann Emerg (Prospective; 18 patients)
Med 1991;20(10):1094-1099. (Retrospective; 166 patients) 121. Sheehan J, White A. Diuretic-associated hypomagnesemia. Brit Med J
98. Heywood JT. Calcium channel blockers for heart rate control in atrial Clin Research Ed 1982;285(6349):1157-1159. (Case series; 21 patients)
fibrillation complicated by congestive heart failure. Can J Cardiol 122. Falcone RA, Morady F, Armstrong WF. Transesophageal
1995;11(9):823-826. (Review) echocardiographic evaluation of left atrial appendage function and
99. Materne P, Legrand V, Vandormael M, et al. Hemodynamic effects of spontaneous contrast formation after chemical or electrical
intravenous diltiazem with impaired left ventricular function. Am J cardioversion of atrial fibrillation. Am J Cardiol 1996;78(4):435-439.
Cardiol 1984;54:733-737. (Prospective, observational; 8 patients) (Prospective; 23 patients)
100. Heywood JT, Graham B, Marais GE, et al. Effects of intravenous 123.* Weigner MJ, Caulfield TA, Danias PG, et al. Risk for clinical
diltiazem on rapid atrial fibrillation accompanied by congestive heart thromboembolism associated with conversion to sinus rhythm in
failure. Am J Cardiol 1991;67:1150-1152. (Prospective, nonblinded, patients with atrial fibrillation lasting less than 48 hours. Ann Intern
uncontrolled; 9 patients) Med 1997;126:615-620. (Prospective; 357 patients)
101. Platia EV, Michelson EL, Porterfield JK, et al. Esmolol versus verapamil 124. Stoddard MF, Dawkins PR, Prince CR, et al. Left atrial appendage
in acute treatment of atrial fibrillation or atrial flutter. Am J Cardiol thrombus is not uncommon in patients with acute atrial fibrillation
1989;63:925-929. (Prospective, randomized, parallel, open-label; 45 and a recent embolic event: a transesophageal echocardiographic
patients) study. J Am Coll Cardiol 1995;25:452-459. (Comparative; 317 patients)
102. Shettigar UR, Toole JG, Appunn DO. Combined use of esmolol and 125. Stoddard MF. Risk of thromboembolism in new onset or transient
digoxin in the acute treatment of atrial fibrillation or flutter. Am Heart J atrial fibrillation. Prog Cardiovasc Dis 1996;39(1):69-80. (Review)
1993;126:368-374. (Prospective; 21 patients) 126. Harenberg J, Huhle G, Piazolo L, et al. Long-term anticoagulation of
103. Moose AN, Wurdeman RL, Mohiuddin SM, et al. Esmolol versus outpatients with adverse events to oral anticoagulants using low-
diltiazem in the treatment of postoperative atrial fibrillation/flutter molecular-weight heparin. Semin Thromb Hemost 1997;23(2):167-172.
after open heart surgery. Am Heart J 2000;140(1):176-180. (Prospective, (Prospective; 10 patients with AF)
randomized; 30 patients) 127. Harenberg J, Weuster B, Pfitzer M, et al. Prophylaxis of embolic events
104. Falk RH, Leavitt JL. Digoxin for atrial fibrillation: a drug whose time in patients with atrial fibrillation using low-molecular-weight heparin.
has gone? Ann Intern Med 1991;114(7):573-575. (Review) Semin Thromb Hemost 1993;19(suppl 1):116-121. (Prospective,
105.* Digitalis in Acute Atrial Fibrillation (DAAF) Trial Group. Intravenous randomized, controlled; 75 patients)
digoxin in acute atrial fibrillation: results of a randomized, placebo- 128. Hollander J. Atrial fibrillation and hospital admission. Acad Emerg Med
controlled multicentre trial in 239 patients. Eur Heart J 1997;18:649-654. 1996;3(8):819-820. (Editorial)
(Prospective, 239 patients) 129. Koenig BO, Ross MA, Jackson RE. An emergency department
106. Tieleman RG, Blaauw Y, Van Gelder IC, et al. Digoxin delays recovery observation unit protocol for acute-onset atrial fibrillation is feasible.
from tachycardia-induced electrical remodeling of the atria. Circulation Ann Emerg Med 2002;39(4):374-381. (Case series; 67 patients)
1999;100(17):1836-1842. (Animal study) 130. Prystowsky EN. Cardioversion of atrial fibrillation to sinus rhythm:
107. Connolly SJ. Evidence based analysis of amiodarone efficacy and who, when, how, and why? Am J Cardiol 2000;86(3):326-327. (Editorial)
safety. Circulation 1999;100(19):2025-2034. (Review) 131. Mandel WJ. Should every patient with atrial fibrillation have the
108. Hou ZY, Chang MS, Chen CY, et al. Acute treatment of recent-onset rhythm converted to sinus rhythm? Clin Cardiol 1994;17(suppl II):II-16-
atrial fibrillation and flutter with a tailored dosing regimen of II-20. (Review)
intravenous amiodarone: a randomized, digoxin controlled study. Eur 132. Hohnloser SH, Kuck KH, Lilienthal J. Rhythm or rate control in atrial
Heart J 1995;16:521-528. (Prospective, open label; 50 patients) fibrillation—pharmacological intervention in atrial fibrillation (PIAF):
109. Vardas PE, Kochiadakis GE, Igoumenidis NE, et al. Amiodarone as a a randomized trial. Lancet 2000;356(9244):1789-1794. (Prospective; 252
first-choice drug for restoring sinus rhythm in patients with atrial patients)
fibrillation: a randomized, controlled trial. Chest 2000;117(6):1538-1545. 133. The Planning and Steering Committees of the AFFIRM Study for the
(Prospective; 208 patients) NHLBI AFFIRM Investigators. Atrial fibrillation follow-up investiga-
110. Hohnloser SH, Klingenheben T, Singh BN. Amiodarone-associated tion of rhythm management-the AFFIRM study design. Am J Cardiol
proarrhythmic effects: a review with special reference to torsades de 1997;79:1198-1202. (Study design)
pointes tachycardia. Ann Intern Med 1994;121(7):529-535. (Review) 134. Stambler BS, Wood MA, Ellenbogen KA, et al. Efficacy and safety of
111. Kochiadakis GE, Igoumenidis NE, Solomou MC, et al. Efficacy of repeated intravenous doses of ibutilide for rapid conversion of atrial
amiodarone for the termination of persistent atrial fibrillation. Am J flutter or fibrillation. Circulation 1996;94(7):1613-1621. (Prospective; 266
Cardiol 1999;83(1):58-61. (Prospective; 67 patients) patients)
112. Noc M, Stajer D, Horvat M. Intravenous amiodarone versus verapamil 135. Volgman AS, Carberry PA, Stambler B, et al. Conversion efficacy and
for acute conversion of paroxysmal atrial fibrillation to sinus rhythm. safety of intravenous ibutilide compared with intravenous
Am J Cardiol 1990;65:679-680. (Prospective, cross-over; 24 patients) procainamide in patients with atrial flutter or fibrillation. J Am Coll
113. Joseph AP, Ward MR. A prospective, randomized controlled trial Cardiol 1998;31:1414-1419. (Prospective, randomized, double-blinded;
comparing the efficacy and safety of sotolol, amiodarone, and digoxin 127 patients)
for the reversion of new-onset atrial fibrillation. Ann Emerg Med 136. Abi-Mansour P, Carberry PA, McCowan RJ, et al. Conversion efficacy
2000;36(1):1-9. (Prospective, non-blinded; 120 patients) and safety of repeated doses of ibutilide in patients with atrial flutter
114. Andrivet P, Boubakri E, Dove PJ, et al. A clinical study of amiodarone and atrial fibrillation. Am Heart J 1998;136:632-642. (Prospective,
as a single oral dose in patients with recent-onset atrial placebo-controlled; 250 patients)
tachyarrhythmia. Eur Heart J 1994;15:1396-1402. (Prospective; 45 137. Wang K, Asinger R, Hodges M. Electrocardiograms of Wolff-
patients) Parkinson-White syndrome simulating other conditions. Am Heart J
115. Boriani G, Biffi M, Frabetti L, et al. Ventricular fibrillation after 1996;132(1):152-155. (Review)
intravenous amiodarone in Wolff-Parkinson-White syndrome with 138. Al-Khatib SM, Pritchett ELC. Clinical features of Wolff-Parkinson-
atrial fibrillation. Am Heart J 1996;131(6):1214-1216. (Case report) White syndrome. Am Heart J 1999;138(3, pt1):403-413. (Review)
116. Hays JV, Gilman JK, Rubal BJ. Effect of magnesium sulfate on
ventricular rate control in atrial fibrillation. Ann Emerg Med
1994;24(1):61-64. (Prospective; 15 patients) Physician CME Questions
117. Gullestad L, Birkeland K, Molstad P, et al. The effect of magnesium
versus verapamil on supraventricular arrhythmias. Clin Cardiol 17. When dealing with atrial fibrillation with a rapid
1993;16:429-434. (Prospective, single-blinded; 57 patients)
118.* Moran JL, Gallagher J, Peake SL, et al. Parenteral magnesium sulfate ventricular response and a wide complex QRS with a
versus amiodarone in the therapy of atrial tachyarrhythmias: a delta wave, which of the following drugs is the safest
prospective, randomized study. Crit Care Med 1995;23(11):1816-1824. to use?
(Prospective; 42 patients)
119. Chiladakis JA, Stathopoulos C, Davlouros P, et al. Intravenous a. Digoxin
magnesium sulfate versus diltiazem in paroxysmal atrial fibrillation. b. Procainamide
Internat J Cardiol 2001;79(2-3):287-91. (Prospective, randomized c. Esmolol
controlled; 46 patients)
120. Brodsky MA, Orlov MV, Capparelli EV, et al. Magnesium therapy in d. Diltiazem
new-onset atrial fibrillation. Am J Cardiol 1994;73:1227-1229. e. Verapamil

Emergency Medicine Practice 26 www.empractice.net • August 2002


18. When using calcium as pretreatment to calcium- 24. Digoxin:
channel blockers, which of the following is true? a. is faster than verapamil for rate control in atrial
a. Calcium will block the chronotropic effects fibrillation with a rapid ventricular response.
of the calcium-channel blocker, but not its b. is better than placebo for conversion of
inotropic effects. atrial fibrillation.
b. Side effects may include a feeling of warmth c. is expensive.
or flushing. d. works not only through direct effects on the AV
c. Calcium gluconate has more calcium per amp node, but also has some indirect effects through
than calcium chloride. vagal stimulation.
d. It should not be used unless the patient is e. can be given safely in WPW.
in extremis. 25. Which of the following is true regarding atrial
e. It should never be given to anyone with fibrillation with a rapid ventricular response?
osteoporosis. a. The coronary arteries will have less time to fill.
19. Diltiazem has clearly been shown to be more effica- b. Cardiac output will likely be decreased, especially
cious and has a higher safety profile than verapamil in the elderly.
in any patient with a rapid ventricular response. c. There is a loss of the “atrial kick.”
a. True d. All of the above.
b. False 26. Amiodarone:
20. When considering cardioversion of an unstable a. has a high incidence of torsades de pointes.
patient with atrial fibrillation, which of the b. can be used for rate control and conversion.
following is true? c. is inexpensive.
a. It should never be done due to the risk of d. has zero incidence of hypotension when
thromboembolism. given rapidly.
b. Intubation should precede cardioversion. 27. ß-blockers:
c. Cardioversion should be performed synchronized. a. should not be given to diabetics with atrial
d. 50J would be expected to work more than 80% fibrillation.
of the time. b. are less effective than digoxin for acute rate
e. Atrial flutter generally requires more energy to control when given IV.
convert than atrial fibrillation. c. are less likely to cause bronchospasm than
21. Every patient with NOAF should have cardiac calcium-channel blockers.
enzymes and be admitted to the hospital to rule d. may be useful in patients with atrial fibrillation
out MI. suspected of having acute coronary syndrome.
a. True e. should be given intravenously along with
b. False calcium-channel blockers to have maximal effects.
22. When considering the risk of thromboembolism after 28. Which of the following has/have been shown to be
cardioversion of atrial fibrillation, which of the effective in rate control of atrial fibrillation?
following is true? a. Magnesium
a. The risk of thromboembolism is zero if the patient b. Verapamil
has been properly anticoagulated. c. Diltiazem
b. Patients with hypercoagulability are probably d. Esmolol
more at risk. e. All of the above
c. The risk of thromboembolism after the patient 29. Atrial flutter:
has received three months of anticoagulation a. rarely, if ever, occurs in association with AF.
is 5%-10%. b. has zero risk of thromboembolism when
d. It is clearly without risk if the patient has had converted.
atrial fibrillation for less than 72 hours. c. can mimic other supraventricular tachycardias.
e. A transesophageal echocardiogram is not likely to d. is typically much more difficult to convert than AF.
be as helpful as a transthoracic echocardiogram.
30. Giving an IV calcium-channel blocker:
23. Which of the following is true regarding NOAF? a. is absolutely contraindicated with signs of
a. It is uncommon in the ED. congestive heart failure.
b. Thyrotoxicosis is an extremely common cause. b. can be done safely immediately after a few
c. There is no risk of thromboembolism if conversion doses of IV propranolol are given.
is less than 48 hours. c. will not work if the patient recently
d. It often presents in children. received calcium.
e. Assuming no other disease process or medication d. is a first-line rate control agent in NOAF at
use, the ventricular rate typically seen is around this time.
150 bpm. e. should always be given with magnesium.

August 2002 • www.empractice.net 27 Emergency Medicine Practice


31. Which of the following is true? Physician CME Information
a. All patients with NOAF should have a d dimer This CME enduring material is sponsored by Mount Sinai School of Medicine and
as a part of the workup to rule out PE. has been planned and implemented in accordance with the Essentials and
b. Magnesium has no effect on ventricular rate in Standards of the Accreditation Council for Continuing Medical Education. Credit
may be obtained by reading each issue and completing the printed post-tests
AF with a rapid ventricular response. administered in December and June or online single-issue post-tests
c. Calcium must be given prior to administration of administered at www.empractice.net.
a calcium-channel blocker to anyone with NOAF. Target Audience: This enduring material is designed for emergency medicine
physicians.
d. Approximately 40%-50% of patients with
Needs Assessment: The need for this educational activity was determined by a
NOAF will convert within the first day survey of medical staff, including the editorial board of this publication; review
regardless of treatment. of morbidity and mortality data from the CDC, AHA, NCHS, and ACEP; and
evaluation of prior activities for emergency physicians.
32. Ibutilide:
Date of Original Release: This issue of Emergency Medicine Practice was published
a. is inexpensive. August 1, 2002. This activity is eligible for CME credit through August 1,
b. has no significant side effects. 2005. The latest review of this material was July 11, 2002.
c. is not FDA approved for use in atrial fibrillation. Discussion of Investigational Information: As part of the newsletter, faculty may
be presenting investigational information about pharmaceutical products that
d. may cause torsades de pointes in up to 8%
is outside Food and Drug Administration approved labeling. Information
of patients. presented as part of this activity is intended solely as continuing medical
e. if successful at conversion, will usually enable education and is not intended to promote off-label use of any pharmaceutical
product. Disclosure of Off-Label Usage: This issue of Emergency Medicine Practice
immediate discharge due to the drug’s short
discusses several antiarrhythmic agents used for elective conversion in the ED.
half-life. While several trials report their use, ibutilide is the only IV agent with FDA
approval for this purpose. (See text.)
Faculty Disclosure: In compliance with all ACCME Essentials, Standards, and
Guidelines, all faculty for this CME activity were asked to complete a full
disclosure statement. The information received is as follows: Dr. Slovis is on the
Class Of Evidence Definitions speaker bureau and receives consultation fees and honoraria from Genentech,
Merck, Cor-Key, and Wyeth. Dr. Pollock and Dr. Herbert report no significant
Each action in the clinical pathways section of Emergency Medicine Practice
financial interest or other relationship with the manufacturer(s) of any
receives an alpha-numerical score based on the following definitions.
commercial product(s) discussed in this educational presentation.
Class I • Case series, animal studies, Accreditation: Mount Sinai School of Medicine is accredited by the Accreditation
• Always acceptable, safe consensus panels Council for Continuing Medical Education to sponsor continuing medical
• Definitely useful • Occasionally positive results education for physicians.
• Proven in both efficacy and
Indeterminate Credit Designation: Mount Sinai School of Medicine designates this educational
effectiveness
• Continuing area of research activity for up to 4 hours of Category 1 credit toward the AMA Physician’s
Level of Evidence: • No recommendations until Recognition Award. Each physician should claim only those hours of credit
• One or more large prospective further research actually spent in the educational activity. Emergency Medicine Practice is approved
studies are present (with by the American College of Emergency Physicians for 48 hours of ACEP Category
rare exceptions) Level of Evidence:
1 credit (per annual subscription). Emergency Medicine Practice has been reviewed
• High-quality meta-analyses • Evidence not available
and is acceptable for up to 48 Prescribed credit hours by the American Academy
• Study results consistently • Higher studies in progress
of Family Physicians. Emergency Medicine Practice has been approved for 48
positive and compelling • Results inconsistent,
Category 2B credit hours by the American Osteopathic Association.
contradictory
Class II • Results not compelling Earning Credit: Two Convenient Methods
• Safe, acceptable • Print Subscription Semester Program: Physicians with current and valid
• Probably useful licenses in the United States who read all CME articles during each
Significantly modified from: The
Emergency Medicine Practice six-month testing period, complete the post-
Level of Evidence: Emergency Cardiovascular Care
test and the CME Evaluation Form distributed with the December and June
• Generally higher levels Committees of the American Heart
issues, and return it according to the published instructions are eligible for
of evidence Association and representatives
up to 4 hours of Category 1 credit toward the AMA Physician’s Recognition
• Non-randomized or retrospec- from the resuscitation councils of
Award (PRA) for each issue. You must complete both the post-test and CME
tive studies: historic, cohort, or ILCOR: How to Develop Evidence-
Evaluation Form to receive credit. Results will be kept confidential. CME
case-control studies Based Guidelines for Emergency
certificates will be delivered to each participant scoring higher than 70%.
• Less robust RCTs Cardiac Care: Quality of Evidence
• Results consistently positive and Classes of Recommendations; • Online Single-Issue Program: Physicians with current and valid licenses in
also: Anonymous. Guidelines for the United States who read this Emergency Medicine Practice CME article
Class III cardiopulmonary resuscitation and and complete the online post-test and CME Evaluation Form at
• May be acceptable emergency cardiac care. Emergency www.empractice.net are eligible for up to 4 hours of Category 1 credit
• Possibly useful Cardiac Care Committee and toward the AMA Physician’s Recognition Award (PRA). You must complete
• Considered optional or Subcommittees, American Heart both the post-test and CME Evaluation Form to receive credit. Results will
alternative treatments Association. Part IX. Ensuring be kept confidential. CME certificates may be printed directly from the Web
Level of Evidence: effectiveness of community-wide site to each participant scoring higher than 70%.
• Generally lower or intermediate emergency cardiac care. JAMA
1992;268(16):2289-2295. Emergency Medicine Practice is not affiliated with any
levels of evidence
pharmaceutical firm or medical device manufacturer.

Publisher: Robert Williford. Executive Editor: Heidi Frost.

Direct all editorial or subscription-related questions to EB Practice, LLC: 1-800-249-5770 • Fax: 1-770-500-1316 • Non-U.S. subscribers, call: 1-678-366-7933
EB Practice, LLC • 305 Windlake Court • Alpharetta, GA 30022
E-mail: emp@empractice.net • Web Site: http://www.empractice.net
Emergency Medicine Practice (ISSN 1524-1971) is published monthly (12 times per year) by EB Practice, LLC, 305 Windlake Court, Alpharetta, GA 30022. Opinions expressed are not necessarily
those of this publication. Mention of products or services does not constitute endorsement. This publication is intended as a general guide and is intended to supplement, rather than
substitute, professional judgment. It covers a highly technical and complex subject and should not be used for making specific medical decisions. The materials contained herein are not
intended to establish policy, procedure, or standard of care. Emergency Medicine Practice is a trademark of EB Practice, LLC. Copyright 2002 EB Practice, LLC. All rights reserved. No part of this
publication may be reproduced in any format without written consent of EB Practice, LLC. Subscription price: $249, U.S. funds. (Call for international shipping prices.)

Emergency Medicine Practice 28 www.empractice.net • August 2002

You might also like